MedSurg Final Exam

Ace your homework & exams now with Quizwiz!

A patient who has a neurologic disease that affects the pyramidal tract is likely to manifest which of the following signs? Impaired muscle movement Decreased deep tendon reflexes Decreased level of consciousness Impaired sensation of touch, pain, and temperature

Impaired muscle movement - Among the most important descending tracts are the corticobulbar and corticospinal tracts, collectively termed the pyramidal tract. These tracts carry volitional (voluntary) impulses from the cortex to the cranial and peripheral nerves. Dysfunction of the pyramidal tract is likely to manifest as impaired movement because of hypertonicity. Diseases affecting the pyramidal tract do not result in changes in LOC, impaired reflexes, or decreased sensation.

Which condition causes heart failure after a myocardial infarction (MI)? Increased workload of the heart. Increased oxygen demands of the heart. Inability of the heart chambers to adequately fill. Impairment of contractile function of the damaged myocardium.

Impairment of contractile function of the damaged myocardium

A patient is admitted to the hospital for evaluation and treatment of thrombocytopenia. Which action is most important for the nurse to implement? Taking the temperature every 4 hours to assess for fever Maintaining the patient on strict bed rest to prevent injury Monitoring the patient for headaches, vertigo, or confusion Removing the oral crusting and scabs with a soft brush four times a day

Monitoring the patient for headaches, vertigo or confusion Rational: The major complication of thrombocytopenia is hemorrhage, and it may occur in any area of the body. Cerebral hemorrhage may be fatal, and evaluation of mental status for CNS alterations to identify CNS bleeding is very important. Fever is not a common finding in thrombocytopenia. Protection from injury to prevent bleeding is an important nursing intervention, but strict bed rest is not indicated. Oral care is performed very gently with minimal friction and soft swabs.

Which nursing action will the nurse include in the plan of care when admitting a patient with an exacerbation of inflammatory bowel disease (IBD)? Restrict oral fluid intake. Monitor stools for blood. Increase dietary fiber intake. ambulate four times daily.

Monitor stools for blood.

Which nursing action will be included in the plan of care for a patient who is being treated for bleeding esophageal varices with balloon tamponade? • Monitor the patient for shortness of breath. • Encourage the patient to cough every 4 hours. • Deflate the gastric balloon every 8 to 12 hours. • Verify the position of the balloon every 6 hours.

Monitor the patient for SOB

Computed tomography of a 68-year-old patient's head reveals that he has experienced a hemorrhagic stroke. Which of the following is a nursing priory intervention in the emergency department? a. maintain the patients airway b. positioning to promote cerebral perfusion c. control fluid and electrolyte electrolyte imbalances d. administration of tissue plasminogen activator (tPA)

a. maintain the patients airway

A patient seen in the asthma clinic has recorded daily peak flows that are 75% of the baseline. Which action will the nurse plan to take next? Increase the dose of the leukotriene inhibitor. Teach the patient about the use of oral corticosteroids. Administer a bronchodilator and recheck the peak flow. Instruct the patient to keep the next scheduled follow-up appointment.

administer a bronchodilator and recheck the peak flow

Which of the following measures should the nurse prioritize when providing care for a patient with a diagnosis of multiple sclerosis (MS)? • Vigilant infection control and adherence to standard precautions • Careful monitoring of neurologic vital signs and frequent reorientation • Maintenance of a calorie count and hourly assessment of intake and output • Assessment of blood pressure and monitoring for signs of orthostatic hypotension

• Vigilant infection control and adherence to standard precautions

A male patient with a diagnosis of Parkinson's disease (PD) has been admitted recently to a long-term care facility. Which of the following actions should the health care team take in order to promote adequate nutrition for this patient? • Provide multivitamins with each meal. • Provide a diet that is low in complex carbohydrates and high in protein. • Provide small, frequent meals throughout the day that are easy to chew and swallow. • Provide the patient with a minced or pureed diet that is high in potassium and low in sodium.

• Provide small, frequent meals throughout the day that are easy to chew and swallow.

The nurse is providing care for a patient who has been diagnosed with Guillain-Barré syndrome. Which of the following assessments should the nurse prioritize? • Pain assessment • Glasgow Coma Scale • Respiratory assessment • Musculoskeletal assessment

• Respiratory assessment

The nurse is caring for a patient admitted with a spinal cord injury following a motor vehicle accident. The patient exhibits a complete loss of motor, sensory, and reflex activity below the injury level. The nurse recognizes this condition as which of the following? • Central cord syndrome • Spinal shock syndrome • Anterior cord syndrome • Brown-Séquard syndrome

• Spinal shock syndrome

The nurse is caring for a patient admitted 1 week ago with an acute spinal cord injury. Which of the following assessment findings would alert the nurse to the presence of autonomic dysreflexia? • Tachycardia • Hypotension • Hot, dry skin • Throbbing headache

• Throbbing headache

Which of the following interventions should the nurse perform in the acute care of a patient with autonomic dysreflexia? • Urinary catheterization • Administration of benzodiazepines • Suctioning of the patient's upper airway • Placement of the patient in the Trendelenburg position

• Urinary catheterization

The nurse is assessing the recent health history of a 63-year-old patient with osteoarthritis (OA). The nurse determines that the patient is trying to manage the condition appropriately when the patient describes which of the following activity patterns? Bed rest with bathroom privileges Daily high-impact aerobic exercise A regular exercise program of walking Frequent rest periods with minimal exercise

A regular exercise program of walking

Cobalamin injections have been prescribed for a patient with chronic atrophic gastritis. The nurse determines that teaching regarding the injections has been effective when the patient states, "The cobalamin injections will prevent me from becoming anemic." "These injections will increase the hydrochloric acid in my stomach." "These injections will decrease my risk for developing stomach cancer." "The cobalamin injections need to be taken until my inflamed stomach heals."

"The cobalamin injections will prevent me from becoming anemic."

While completing an admission history for a 73-year-old man with osteoarthritis admitted for knee arthroplasty, the nurse asks about the patient's perception of the reason for admission. The nurse expects the patient to relate which of the following responses to this question? Recent knee trauma Debilitating joint pain Repeated knee infections Onset of "frozen" knee joint

Debilitating joint pain

After a patient has had a transjugular intrahepatic portosystemic shunt (TIPS) placement, which finding indicates that the procedure has been effective? • Lower indirect bilirubin level • Increase in serum albumin level • Decrease in episodes of variceal bleeding • Improvement in alertness and orientation

Decrease in episodes of variceal bleeding

The nurse is planning care for a patient who is diagnosed with peripheral vascular disease (PVD) and has a history of heart failure. The nurse should develop a plan of care that is based on the fact that the patient may have a low tolerance for exercise related to: Decreased blood flow. Increased blood flow. Decreased pain. Increased blood viscosity.

Decreased blood flow

The nurse is caring for a patient who has returned to the unit following the application of a cast for a fracture of the right ulna. The patient is now complaining of severe pain, numbness, and tingling of the right arm. What is the most important action of the nurse? a. Administer acetaminophen (Tylenol) as prescribed. Lower the arm below the level of the heart. Immediately report the patient's symptoms. Apply a heating pad.

Immediately report the patient's symptoms.

The nurse is caring for a 49-year-old patient admitted to the nursing unit with osteomyelitis. Which of the following symptoms will the nurse most likely find on physical examination of the patient? Nausea and vomiting Localized pain and redness Paresthesia in the affected extremity Generalized bone pain throughout the leg

Localized pain and redness

A homeless patient with severe anorexia and fatigue is admitted to the hospital with viral hepatitis. Which patient goal has the highest priority when the nurse is developing the plan of care? • Increase activity level. • Maintain adequate nutrition. • Establish a stable home environment. • Identify the source of exposure to hepatitis.

Maintain adequate nutrition

The nurse is completing discharge teaching with an 80-year-old male patient who underwent right total hip arthroplasty. The nurse identifies a need for further instruction if the patient states the need to do which of the following? Avoid crossing his legs. Use a toilet elevator on toilet seat. Notify future caregivers about the prosthesis. Maintain hip in adduction and internal rotation.

Maintain hip in adduction and internal rotation.

The nurse is teaching a class on primary prevention of osteoporosis. What is the most important information for the nurse to provide? Maintain the optimal calcium intake. Place items within reach of the patient. Install bars in the bathroom to prevent falls. Use a professional alert system in the home in case a fall occurs when the patient is alone.

Maintain the optimal calcium intake.

Which of the following is a priority goal for the patient with chronic obstructive pulmonary disease (COPD)? Maintaining functional ability Minimizing chest pain. Increasing carbon dioxide levels in the blood. Treating infectious agents.

Maintaining functional ability

A patient with heart failure says he sleeps with two pillows because he experiences difficulty breathing when lying flat. The nurse would document this as which condition? Bradypnea Dyspnea on exertion Paroxysmal nocturnal dyspnea Orthopnea

Orthopnea

A patient asks the nurse, "What is the difference between rheumatoid arthritis and osteoarthritis?" What is the most appropriate response by the nurse? Osteoarthritis is gender specific: rheumatoid arthritis is not. Osteoarthritis is a systemic disease: rheumatoid arthritis is localized. Osteoarthritis is a localized disease: rheumatoid arthritis is systemic. Osteoarthritis has dislocation and subluxations; rheumatoid arthritis does not.

Osteoarthritis is a localized disease: rheumatoid arthritis is systemic.

Which of the following are causes of primary osteoarthritis? Overuse of joints, aging, and obesity Obesity, aging, and diabetes mellitus Congenital abnormality, aging, overuse of joints Diabetes mellitus, congenital abnormality, aging

Overuse of joints, aging, and obesity

The nurse is admitting a patient who is scheduled for knee arthroscopy related to osteoarthritis (OA). Which of the following findings would the nurse expect to be present on examination of the patient's knees? Ulnar drift Pain with joint movement Reddened, swollen affected joints Stiffness that increases with movement

Pain with joint movement

A patient has returned from the cardiac catheterization laboratory after a balloon valvuloplasty for mitral stenosis.. which of the following requires immediate nursing action? A low, grade intensity mitral regurgitation murmur. SpO2 is 94% on 2 L of oxygen via nasal cannula. The patient has become more somnolent. Urine output has decreased from 80 to 60 mL over the last hour.

The patient has become more somnolent

When assessing a patient with osteoarthritis, which of the following instructions would be considered primary prevention of injury from osteoarthritis? Select all that apply. The patient should avoid repetitive tasks. The patient should avoid physical activity. The patient should warm up before exercising. The patient should perform only repetitive tasks. The patient should limit periods of activity to the morning hours.

The patient should avoid repetitive tasks. The patient should warm up before exercising.

A patient with peripheral vascular disease has chronic severe pretibial and ankle edema bilaterally. Because the patient is on complete bed rest and circulation is compromised, one goal is to maintain tissue integrity. Which of the following interventions will help achieve this outcome? a. Administering pain medications b. Encouraging fluids c. Turning the patient every 1 to 2 hours. d. maintaining hygiene.

Turn the patient every 1 to 2 hours

Which information will be best for the nurse to include when teaching a patient with peptic ulcer disease (PUD) about dietary management of the disease? "Avoid foods that cause pain after you eat them." "High-protein foods are least likely to cause pain." "You will need to remain on a bland diet indefinitely." "you should avoid eating many raw fruits and vegetables."

"Avoid foods that cause pain after you eat them."

A patient with a hematologic disorder has a smooth, shiny, red tongue. Which laboratory result would the nurse expect to see? Neutrophils 45% Hgb 9.6 g/dL WBC count 13,500/uL Red blood cell (RBC) count 6.4S10uL

hgb 9.6 WBC: 4500-11,000 Neutropphil 2500-8000 Hgb 14-18 RBC 4.7-6.1

Which of the following findings would suggest pneumothorax in a trauma victim? Pronounced crackles. Inspiratory wheezing. Dullness on percussion. Absent breath sounds.

Absent breath sounds

Musculoskeletal assessment is an important component of care for patients on long-term therapy of Corticosteroids. Antiplatelet aggregators. Adrenergic blockers. Calcium-channel blockers.

Corticosteroids

A nurse is caring for a patient with cardiomyopathy and is aware that the patient is at high risk for developing: heart failure. diabetes mellitus. myocardial infarction (MI). pericardial effusion.

Heart failure

The nurse teaches a patient who has asthma about peak flow meter use. Which action by the patient indicates that teaching was successful? The patient inhales rapidly through the peak flow meter mouthpiece. The patient takes montelukast (Singulair) for peak flows in the red zone. The patient calls the health care provider when the peak flow is in the green zone. The patient uses albuterol (Proventil) metered dose inhaler (MDI) for peak flows in the yellow zone.

The patient uses albuterol metered dose inhaler for peak flows in the yellow zone

Which of the following interventions should the nurse anticipate in a client who has been diagnosed with acute respiratory distress syndrome (ARDS)? Tracheostomy. Use of a nasal cannula. Mechanical ventilation. Insertion of a chest tube.

mechanical ventilation

A nurse is caring for a patient with cardiomyopathy and is aware that the patient is at high risk for developing: a. heart failure. b. diabetes mellitus. c. myocardial infarction (MI). d. pericardial effusion.

A

Which of the following nursing diagnoses is a priority in the care of a patient with renal calculi? a. Acute pain b. Deficient fluid volume c. Risk for constipation d. Risk for powerlessness

a. Acute pain

A patient has just arrived on the postoperative unit after having a laparoscopic esophagectomy for treatment of esophageal cancer. Which nursing action should be included in the postoperative plan of care? a. Elevate the head of the bed to at least 30 degrees. b. Reposition the nasogastric (NG) tube if drainage stops or decreases. c. Notify the doctor immediately about bloody NG drainage. D. Start oral fluids when the patient has active bowel sounds.

a. Elevate the head of the bed to at least 30 degrees.

Which of the following modifiable risk factors for stroke would be most important for the nurse to include when planning a community education program? a. hypertension b. hyperlipidemia c. alcohol consumption d. oral contraceptive use

a. hypertension

The patient has an order for phenytoin 100mg q8hr IV. Available is a phenytoin injection containing 50 mg/ml. How many millimeters of solution should the nurse draw up for the dose? a. 0.5 b. 2 c. 5 d. 20

b. 2

A patient has been admitted with endocarditis. The patient asks the nurse how the health care provider knows what medicines to order to kill the bacteria. The nurse knows that to find the most appropriate antibiotic treatment, what test would be most helpful? a. Echocardiogram b. Blood cultures c. Arterial blood gases (ABGs) d. Complete blood count (CBC)

b. Blood cultures

A female patient has left-sided hemiplegia following an ischemic stroke that she experienced 2 weeks earlier. How should the nurse best promote the health of the patient's integumentary system? a. position the patient on her weak side the majority of the time b. alternate the patient's positioning between supine and side-laying c. avoid the use of pillows in order to promote independence in position d. establish a schedule for massage of areas where skin breakdown emerges

b. alternate the patient's positioning between supine and side-laying

The nurse would expect to find which of the following clinical manifestations in a patient admitted with a left-brain stroke? a. impulsivity b. impaired speech c. left sided neglect d. short attention span

b. impaired speech

The patient has been receiving scheduled doses of phenytoin and begins to experience diplopia. The nurse immediately assess the patient for which of the following? a. an aura b. nystagmus or confusion c. abdominal pain or cramping d. irregular pulse or palpitations

b. nystagmus or confusion

A 16 year old girl presents at the emergency department complaining of right lower quadrant pain and is diagnosed with appendicitis. when assessing this patient. what signs or symptoms should the nurse expect to find? a. rigid abdomen, levine's sign, pain relief leaning forward b. rebound tenderness, McBurney's sign, lower grade fever c. Right lower quadrant pain, Ckvostek's sign, muscle guarding d. Preumqbicical pain, Trousseau's sign, pain relief with pressure

b. rebound tenderness, McBurney's sign, lower grade fever

A 70-year-old female patient complains of lower back pain and is diagnosed with osteoporosis. The nurse is aware that this patient is most at risk for which condition? Pain Fracture Harding of the bones Increased bone matrix and remineralization

fracture

A patient's sudden onset of hemiplegia has necessitated a computed tomography (CT) of her head. Which of the following assessments should the nurse complete prior to this diagnostic study? a. assess the patients immunization history b. screen the patient for any metal parts or a pacemakers c. assess the patients for allergies to shellfish, iodine, or dyes d. assess the patients need for tranquilizers or anti-seizure medications

c. assess the patients for allergies to shellfish, iodine, or dyes

Which of the following sensory-perceptual deficits is associated with left-brain stroke (right hemiplegia)? a. overestimation of physical abilities b. difficulty judging position and distance c. slow and possibly fearful performance of tasks d. impulsivity and impatience at performing tasks

c. slow and possibly fearful performance of tasks

The health care provider orders lactulose for a patient with hepatic encephalopathy. The nurse will monitor for effectiveness of this medication for this patient by assessing which of the following? • Relief of constipation • Relief of abdominal pain • Decreased liver enzymes • Decreased ammonia levels

decreased ammonia levels

A patient from a Mediterranean country is admitted with thalassemia, jaundice, splenomegaly, and hepatomegaly. Which of the following should be the primary focus of nursing care for this client? Providing activities of daily living on the same schedule of the patient's homeland. Offering foods that the patient enjoys in order to increase the intake of calories. Decreasing cardiac demands by promoting rest. Listening to concerns about the hospitalization.

decreasing cardiac demands by promoting rest

A nurse is assessing a female who is receiving her second administration of chemotherapy for breast cancer. When obtaining this patient's health history, what is the most important information the nurse should obtain? "Has your hair been falling out in clumps?" "Have you had nausea or vomiting?" "Have you been sleeping at night?" "Do you have your usual energy level?"

"Have you had nausea or vomiting?"

A 62-year-old patient who has been diagnosed with esophageal cancer tells the nurse, "I know that my changes are not very good, but I do not feel ready to die yet." Which response by the nurse is most appropriate? "You may have quite a few years still left to live." "Thinking about dying will only make you feel worse." "Having this new diagnosis must be very hard for you." "It is important that you be realistic about your prognosis."

"Having this new diagnosis must be very hard for you."

You provide patient education related to the management of iron deficiency anemia. Which of the following statements made by the patient signifies understanding of the education provided? "I should take my iron pills with a glass of orange juice." "I should take my iron pills with breakfast to decrease stomach upset." "Iron pills often cause constipation, so I should decrease by fluid and fiber intake." "I will only need to take these pills for a few days and then my problem will be fixed."

"I should take my iron pills with a glass of orange juice."

When reinforcing health teaching about the management of osteoarthritis (OA), the nurse determines that the patient needs additional instruction after making which of the following statements? "I should take the Celebrex as prescribed to help control the pain." "I should try to stay standing all day to keep my joints from becoming stiff." "I can use a cane if I find it helpful in relieving the pressure on my back and hip." "A warm shower in the morning will help relieve the stiffness I have when I get up."

"I should try to stay standing all day to keep my joints from becoming stiff."

A 54-year-old patient with acute osteomyelitis asks the nurse how this problem will be treated. Which of the following responses by the nurse is most appropriate? "Oral antibiotics are often required for several months." "Intravenous antibiotics are usually required for several weeks." "Surgery is almost always necessary to remove the dead tissue that is likely to be present." "Drainage of the foot and instillation of antibiotics into the affected area is the usual therapy."

"Intravenous antibiotics are usually required for several weeks."

Which information will the nurse include when teaching a patient with newly diagnosed gastroesophageal reflux disease (GERD)? "Peppermint tea may be helpful in reducing your symptoms." "you should avoid eating between meals to reduce acid secretion." "Vigorous physical activities may increase the incidence of reflux." "It will be helpful to keep the head of your bed elevated."

"It will be helpful to keep the head of your bed elevated."

The results of a patient's recent endoscopy indicate the presence of peptic ulcer disease (PUD). Which of the following teaching points should the nurse provide to the patient in light of his new diagnosis? "You'll need to drink at least two to three glasses of milk daily." "It would likely be beneficial for you to eliminate dringing alcohol." "Many people find that a minced or pureed diet eases their symptoms of PUD>" "Your medications should allow you to maintain your present diet while minimizing symptoms."

"It would likely be beneficial for you to eliminate dringing alcohol."

The incidence and risk of cancer increase when smoking is combined with: Asbestos exposure and alcohol consumption. Ultraviolet radiation exposure and alcohol consumption. Asbestos exposure and ultraviolet radiation exposure. Alcohol consumption and human papilloma virus (HPV) infection.

Asbestos exposure and alcohol consumption.

The patient diagnosed with cancer is receiving chemotherapy. The nurses should assess which of the following diagnostic values while the client is receiving chemotherapy? Bone marrow cells Liver tissues Heart tissues Pancreatic enzymes

Bone marrow cells

Which of the following clinical manifestations would the nurse interpret as representing neurogenic shock in a patient with acute spinal cord injury? Bradycardia Hypertension Neurogenic spasticity Bounding pedal pulses

Bradycardia Neurogenic shock is due to the loss of vasomotor tone caused by injury and is characterized by bradycardia and hypotension. Loss of sympathetic innervations causes peripheral vasodilation, venous pooling, and a decreased cardiac output. Thus hypertension, neurogenic spasticity, and bounding pedal pulses are not seen in neurogenic shock.

The nurse is preparing an educational program on breast cancer for women at an African American community center. What information is important for the nurse to consider for the discussion? African American women have the lowest rate of breast cancer Most African American women are diagnosed early in the disease process. Breast cancer concerns vary between socioeconomic levels of African American women. African American women believe breast cancer is inevitable.

Breast cancer concerns vary between socioeconomic levels of african american women

After receiving change-of-shift report, which of the following patients should the nurse assess first? A patient whose new ileostomy has drained 800 mL over the previous 8 hours. A patient with familial adenomatous polyposis who has occult blood in the stool. A patient with ulcerative colitis who has had six liquid stools in the previous 4 hours. A patient who has abdominal distention and an apical heart rate of 136 beats/minute.

A patient who has abdominal distention and an apical heart rate of 136 beats/minute.

A nurse is reviewing the laboratory result of a patient with anemia and anticipates which lab value would be decreased? a. Erythrocytes b. Granulocytes c. Leukocytes d. Platelets

A

A nurse is teaching a patient about cardiomyopathy and determines further teaching is needed when the patient states: a. "It is caused by a plaque in the arteries." b. "It is caused by a virus." c. "It is caused by bacteria." d. "It is caused by certain drugs

A

A patient is admitted with a possible diagnosis of rheumatoid arthritis (RA). Which of the following screening tests should the nurse expect to be ordered? a. Antinuclear antibody (ANA) titer b. Complete blood count (CBC) c. Erythrocyte sedimentation rate (ESR) d. rheumatoid factor (RF)

A

A patient with renal insufficiency is admitted with a diagnosis of pneumonia. He's being treated with I.V. antibiotics, which can be nephrotoxic. Which laboratory value(s) should be monitored closely? a. Blood urea nitrogen (BUN) and creatinine levels b. Arterial blood gas (ABG) levels c. Platelet count d. Potassium level

A

An elderly patient developed pneumonia. The nurse is aware that the initial symptom the patient may manifest is: a. altered mental status and dehydration b. fever and chills c. hemoptysis and dyspnea d. pleuritic chest pain and cough

A

The health care provider prescribed salicylates for a patient with osteoarthritis. The nurse assesses the patient and determines further intervention is necessary when the patient exhibits which of the following? a. Hearing loss b. Increased pain in joints c. Decreased calcium absorption d. Increased bone demineralization

A

The nurse is teaching a class on primary prevention of osteoporosis. What is the most important information for the nurse to provide? a. Maintain the optimal calcium intake. b. Place items within reach of the patient. c. Install bars in the bathroom to prevent falls. d. Use a professional alert system in the home in case a fall occurs when the patient is along.

A

The nurse reviews the assessment of a patient with acute respiratory distress syndrome (ARDS). What is the best indicator of improvement in the patient? a. Arterial blood gas (ABG) values b. Bronchoscopy results c. Increased blood pressure d. Sputum culture and sensitivity results

A

Which of the following are causes of primary osteoarthritis? a. Overuse of joints, aging, and obesity b. Obesity, aging, and diabetes mellitus c. Congenital abnormality, aging, overuse of joints d. Diabetes mellitus, congenital abnormality, aging

A

Which of the following nursing actions should be implemented in the care of a patient who is experiencing increased intracranial pressure (ICP)? Monitor fluid and electrolyte status astutely. Position the patient in a high Fowler's position. Administer vasoconstrictors to maintain cerebral perfusion. Maintain physical restraints to prevent episodes of agitation.

A) Monitor fluid and electrolyte status astutely Fluid and electrolyte disturbances can have an adverse effect on ICP and must be monitored vigilantly. The head of the patient's bed should be kept at 30 degrees in most circumstances, and physical restraints are not applied unless absolutely necessary. Vasoconstrictors are not typically administered in the treatment of ICP.

Magnetic resonance imaging (MRI) has revealed the presence of a brain tumor in a patient. The nurse would recognize the patient's likely need for which of the following treatment modalities? Surgery Chemotherapy Radiation therapy Pharmacologic treatment

A) Surgery Surgical removal is the preferred treatment for brain tumors.

The nurse is caring for a patient receiving digoxin (lanoxin). Which of the following manifestations correlate with a digoxin level of 2.3 ng/dl? Select all that apply. a. nausea b. Drowsiness c. Photophobia d. Increased appetite e. Increased energy level f. Seeing halos around bright objects

A, B, C, F

A patient was admitted with iron deficiency anemia and blood streaked emesis. Which question is most appropriate for the nurse to ask in determining the extent of the patient's activity intolerance? "What daily activities were you able to do 6 months ago compared with the present?" "How long have you had this problem?" "Have you been able to keep up with all your usual activities?" "Are you more tired now than you used to be?"

A It is difficult to determine activity intolerance without comparing from one time frame to another. Iron deficiency anemia can occur gradually and individual endurance varies, ask the pt to compare activities 6 months ago and at the present.

During a public health screening day, which of the following assessment findings would alert the nurse to the presence of osteoporosis in a 61-year-old female? The presence of bowed legs A measurable loss of height Poor appetite and aversion to dairy products The development of unstable, wide-gait ambulation

A measurable loss of height

Which of the following patients with cancer should the nurse assess first? A 38-year old woman receiving internal radiation therapy for cervical cancer. A 27-year old man with leukemia hospitalized for induction of high-dose chemotherapy A 75-year old man with metastatic prostate cancer with a pathologic fracture of the femur who is in pain A 23-year old woman undergoing surgery for placement of a central venous catheter.

A 75-year old man with metastatic prostate cancer with a pathologic fracture of the femur who is in pain

Which of the following patients with cancer should the nurse assess first? A 38-year-old woman receiving internal radiation therapy for cervical cancer. A 27-year-old man with leukemia hospitalized for induction of high-dose chemotherapy. A 75-year-old man with metastatic prostate cancer with a pathologic fracture of the femur who is in pain. A 23-year-old woman undergoing surgery for placement of a central venous catheter.

A 75-year-old man with metastatic prostate cancer with a pathologic fracture of the femur who is in pain.

The nurse is performing an admission assessment on a patient who has been diagnosed with a cardiovascular disease. The nurse would anticipate the data to include which of the following? Select all that apply. a. Fatigue b. Chest Pain c. Weight loss d. Light-headedness e. Dependent edema f. Difficulty breathing in an upright position

A, B, D, E

When assessing a patient with osteoarthritis, which of the following instructions would be considered primary prevention of injury from osteoarthritis? Select all that apply. a. The patient should avoid repetitive tasks b. The patient should avoid physical activity. c. The patient should warm up before exercising. d. The patient should perform only repetitive tasks

A, C

The nurse determines that dietary teaching for a 75-year-old patient with osteoporosis has been most successful when the patient selects which of the following highest-calcium meals? Chicken stir-fry with 1 cup each onions and snap peas, and 1 cup of steamed rice Ham and Swiss cheese sandwich on whole wheat bread, steamed broccoli, and an apple A sardine (3 oz) sandwich on whole wheat bread, 1 cup of fruit yogurt, and 1 cup of skim milk A two-egg omelet with 2 oz of American cheese, one slice of whole wheat toast, and a half grapefruit

A sardine (3 oz) sandwich on whole wheat bread, 1 cup of fruit yogurt, and 1 cup of skim milk

The QRS complex indicates: a. ventricular depolarization b. ventricular redepolarization c. atrial depolarization d. an abnormal conduction pathway

A ventricular depolarization - contraction ventricular depolarization - T wave atrial depolarization - P wave

A patient with a suspected closed head injury has bloody nasal drainage. The nurse suspects that this patient has a cerebrospinal fluid (CSF) leak when observing which of the following? A halo sign on the nasal drip pad Decreased blood pressure and urinary output A positive reading for glucose on a Test-tape strip Clear nasal drainage along with the bloody discharge

A) Halo sign on the nasal drip pad When drainage containing both CSF and blood is allowed to drip onto a white pad, within a few minutes the blood will coalesce into the center, and a yellowish ring of CSF will encircle the blood, giving a halo effect. The presence of glucose would be unreliable for determining the presence of CSF because blood also contains glucose

The nurse is teaching a patient about the risk factors of osteoporosis. It is most important for the nurse to include which factors? Select all that apply. a. Inadequate dietary intake of calcium b. Blood pressure medications c. Family history d. Smoking e. Oral hypoglycemics

A, C, D

The nurse is caring for a patient admitted for evaluation and surgical removal of a brain tumor. The nurse will plan interventions for this patient based on knowledge that brain tumors can lead to which of the following complications (select all that apply)? A. Vision loss B. Cerebral edema C. Pituitary dysfunction D. Parathyroid dysfunction E. Focal neurologic deficits

A,B,C,E Brain tumors can manifest themselves in a wide variety of symptoms depending on location, including vision loss and focal neurologic deficits. Tumors that put pressure on the pituitary can lead to dysfunction of the gland. As the tumor grows, clinical manifestations of increased intracranial pressure (ICP) and cerebral edema can appear. The parathyroid gland is not regulated by the cerebral cortex or the pituitary gland.

A patient is admitted to the emergency department with possible renal trauma after an automobile accident. Which prescribed intervention will the nurse implement first? A. Check blood pressure and heart rate. B. Administer morphine sulfate 4mg IV. C. Transport to radiology for an intravenous pyelogram. D. Insert a urethral catheter and obtain a urine specimen.

A. Check blood pressure and heart rate.

Which medication taken at home by a 47-year-old patient with decreased renal function will be of most concern to the nurse? A. ibuprofen (Motrin) B. warfarin (Coumadin) C. folic acid (vitamin B) D. penicillin (Bicillin LA)

A. ibuprofen (Motrin)

A patient with acute pancreatitis has a nasogastric (NG) tube to suction and is NPO. Which information obtained by the nurse indicates that these therapies have been effective? • Bowel sounds are present. • Grey Turner sign resolves. • Electrolyte levels are normal. • Abdominal pain is decreased.

Abdominal pain is decreased

The primary purpose of the Schilling test is to measure the patient's ability to: store vitamin B12. digest vitamin B12. absorb vitamin B12. produce vitamin B12.

Absorb b12 Pernicious anemia is caused by the body's inability to absorb vitamin B12. This results in a lack of intrinsic factor in the gastric juices. Schilling's test helps diagnose pernicious anemia by determining the its ability to absorb vitamin B12.

When instructing patients on how to decrease the risk of chronic obstructive pulmonary disease (COPD), the nurse should emphasize which of the following? Participate regularly in aerobic exercises. Maintain a high-protein diet. Avoid exposure to people with known respiratory infections. Abstain from cigarette smoking.

Abstain from cigarette smoking

When implementing the initial plan of care for a patient admitted with acute diverticulitis, the nurse will plan to collect a stool specimen. prepare for colonoscopy. schedule a barium enema have blood cultures drawn.

Administer IV fluids collect a stool specimen.

A 54-year-old patient admitted with diabetes mellitus, malnutrition, osteomyelitis, and alcohol abuse has a serum amylase level of 280 U/L and a serum lipase level of 310 U/L. To which of the following diagnoses does the nurse attribute these findings? • Malnutrition • Osteomyelitis • Alcohol abuse • Diabetes mellitus

Alcohol abuse

Sinus tachycardia may be seen as response to: a. anxiety b. pain c. fever d. all of the above

All of the above

Which of the following should be done when preparing and administering insulin for injection ( select all) a. If mixing insulin, draw up short acting insulin first b. Shake vials vigorously prior to drawing up c. Administer injection subcutaneously d. Check blood sugar prior to administration, if ordered e. Draw up the long acting insulin first

Answer: a, c, d

You need to give Mr. Brady his 9am heparin dose (subcutaneous) which needle do you choose? a. 1 ½ - 2 inch needle, with 16 gauge b. ½ inch needle with 20 gauge c. 3/8 - 5/8 inch needle with 25 gauge d. 3/8 - 5/8 needle with 21 gauge

Answer: c

Prior to administering an oral medication, the nurse should evaluate the patient for which of the following? a. The presence of GI alterations b. The inability of the patient to swallow foods or fluids c. The use of gastric suction d. Any contraindication relevant to that medication e. All of the above

Answer: e

1. A legal medication order contains all of the following pieces of information (select all) a. The name of the patient b. The name of the drug c. The dosage and route of the drug d. The name of the person administering the drug e. The provider signature

Answer: A, b, c, e

Mr. B is to receive 5000 units of heparin SC, the best way to administer this medication is: a. In the upper thigh at 90 degree angle, no aspiration b. Heparin always needs to be by Z track administration c. In the abdomen at 90 degree angle, no aspiration d. In the arm at 45 degree angle, with aspiration

Answer: C

Part of your responsibility in the pre-medication process includes: a. Make sure the medication is not beyond the expiration date b. Assessment of labs, vital signs parameters c. Use of aseptic technique, washing your hands d. All of the above

Answer: D

When administering ophthalmic medication, the nurse should (select all) a. Avoid touching the cornea with the eyedropper b. Interpret OS to indicate left eye c. Apply ointment to 1/3 of the lower lid d. Keep eyes closed for 5 minutes after getting medication

Answer: a, b, c

A nurse is providing education in a community setting about general measures to avoid excessive sun exposure. Which of the following recommendations is appropriate? Apply sunscreen only after going into the water. Avoid peak exposure hours from 9 am to 1 pm. Wear loosely woven clothing for added ventilation. Apply sunscreen with a sun protection factor (SPF) of 15 or more before sun exposure.

Apply sunscreen with a sun protection factor (SPF) of 15 or more before sun exposure.

A patient who has had a total proctocolectomy and permanent ileostomy tells the nurse "I cannot bear to even look at the stoma l do not think I can manage all these changes." Which is the best action by the nurse? Develop a detailed written plan for the ostomy care for the patient. Ask the patient more about the concerns with stoma management. Reassure the patient that care for the ileostomy will become easier. Postpone any patient teaching until the patient adjusts to the ileostomy.

Ask the patient more about the concerns with stoma management.

You place electrodes on a collapsed individual who was visiting a hospitalized family member, the monitor exhibits ventricular tachycardia. Which intervention would you do first? Place the patient on oxygen Confirm the rhythm with a 12-lead ECG Administer amiodarone I.V. as prescribed. Assess the patient's airway, breathing, and circulation

Assess the pos airway, breathing and circulation

Your patients request valium (diazepam). You check the medication administration record and find that it was a one-time order, which was administered during the previous shift. Your response is a. Give a second dose and document it accordingly b. Assess your patient and give valium only if BP is higher than 150/90 c. Assess your patient for anxiety and try non pharmacological methods for controlling it, page the MD for another medication order, if necessary d. Give a small dose and notify the MD

Assess your patient for anxiety and try non pharmacological methods for controlling it, page the MD for another medication order, if necessary

When assessing a patient with a traumatic brain injury, the nurse notes uncoordinated movement of the extremities. The nurse would document this as Ataxia. Apraxia. Anisocoria. Anosognosia.

Ataxia* is a lack of coordination of movement, possibly caused by lesions of sensory or motor pathways, cerebellum disorders, or certain medications. Apraxia is the inability to perform learned movements despite having the desire and physical ability to perform them related to a cerebral cortex lesion. Anisocoria is inequality of pupil size from an optic nerve injury. Anosognosia is the inability to recognize a bodily defect or disease related to lesions in the right parietal cortex.

To decrease the risk for cancers of the tongue and buccal mucosa, which information will the nurse include when teaching a patient who is seen for an annual physical exam in the outpatient clinic? Avoid use of cigarettes and smokeless tobacco. Use sunscreen when outside even on cloudy days. Complete antibiotics used to treat throat infections. Use antivirals to treat herpes simplex virus (HSV) infections.

Avoid use of cigarettes and smokeless tobacco.

A 70-year-old female patient complains of lower back pain and is diagnosed with osteoporosis. The nurse is aware that this patient is most at risk for which condition? a. Pain b. Fracture c. Hardening of the bones d. Increased bone matrix and remineralization

B

A nurse on a telemetry unit teaches a patient diagnosed with acute coronary syndrome about coronary blood flow. Which of the following statement made by the nurse is correct. a. Most of the blood flow to coronary arteries is supplied during inspiration. b. Most of the blood flow to coronary arteries is supplied during diastole. c. Blood flow to coronary arteries is related to breathing patterns. d. A large portion of blood flow occurs to coronary arteries during systole.

B

A patient has been treated for compartment syndrome by undergoing a fasciotomy. Which nursing diagnosis has the highest priority for this patient. a. Chronic pain b. Risk for infection c. Impaired gas exchange d. Decreased cardiac output

B

A patient is diagnosed with cystitis. The nurse provides teaching aimed at preventing a recurrence. Which instruction does the nurse provide to the patient? a. Bathe in a tub. b. Wear cotton underwear. c. Use a feminine hygiene spray. d. Limit your intake of cranberry juice.

B

A patient is experiencing signs and symptoms of coronary artery disease. What should be the nurse's first priority? a. Decrease anxiety. b. enhance myocardial oxygenation. c. Administer sublingual nitroglycerin. d. Educate the patient about his symptoms.

B

A patient is scheduled for magnetic resonance imaging (MRI) of the head. Which area is essential to assess before the procedure. a. Food or drink intake within the past 8 hours. b. Prostheses or a pacemaker c. The presence of carotid artery disease d. Voiding before the procedure

B

A patient with acute renal failure has a serum potassium level of 7.0mEq/L. the nurse's priority for this patient is to assess which of the following? a. Urine specific gravity b. Electrocardiogram (ECG) results c. Mental status d. Blood pressure

B

The nurse is reviewing a 52-year-old patient's laboratory values. The platelet count is 75,000//ul. How would you interpret this value? a. Normal platelet count b. Thrombocytopenia c. Thrombocytopathy d. Thrombocytosis

B

The nurse is reviewing a patient's complete blood count and notes a decreased number of erythrocytes, leukocytes, and platelets. The nurse interprets this as indicative of what condition? a. Pernicious anemia b. Aplastic anemia c. Sickle cell anemia d. Polycythemia

B

What is the first intervention for the nurse to implement for a patient experiencing myocardial infarction (MI)? a. Administer morphine b. Administer oxygen c. Administer sublingual nitroglycerin d. Obtain an electrocardiogram (ECG)

B

When assessing the ability of a patient's pupil to constrict, which cranial nerve (CN) is being tested? a. II b. III c. IV d. V

B

A patient's electrocardiogram shows ST elevation in leads II, III, and aVf suggesting occlusion of the right coronary artery. The patient asks the nurse what area of the heart this has affected? What is the nurse's best response? a. Anterior b. apical c. Inferior d. Lateral

C

How should the nurse most accurately assess the position sense of a patient with a recent traumatic brain injury? Ask the patient to close his or her eyes and slowly bring the tips of the index fingers together. Ask the patient to maintain balance while standing with his or her feet together and eyes closed. Ask the patient to close his or her eyes and identify the presence of a common object on the forearm. Place the two points of a calibrated compass on the tips of the fingers and toes and ask the patient to discriminate the points.

B The Romberg test is an assessment of position sense in which the patient stands with the feet together and then closes his or her eyes while attempting to maintain balance. The other cited tests of neurologic function do not directly assess position sense.

Parasympathetic stimulation of the heart can a. increase the heart rate b. decrease the heart rate c. increase contractility of the heart d. no effect

B Decrease the heart rate

When assessing the accessory nerve, the nurse would Assess the gag reflex by stroking the posterior pharynx. Ask the patient to shrug the shoulders against resistance. Ask the patient to push the tongue to either side against resistance. Have the patient say "ah" while visualizing elevation of soft palate.

B shoulder against resistance - The spinal accessory nerve is tested by asking the patient to shrug the shoulders against resistance and to turn the head to either side against resistance while observing the sternocleidomastoid muscles and the trapezius muscles. Assessing the gag reflex and saying "ah" are used to assess the glossopharyngeal and vagus nerves. Asking the patient to push the tongue to either side against resistance and to stick out the tongue are used to assess the hypoglossal nerve.

The nurse is caring for a patient admitted with a subdural hematoma following a motor vehicle accident. Which of the following changes in vital signs would the nurse interpret as a manifestation of increased intracranial pressure? Tachypnea Bradycardia Hypotension Narrowing pulse pressure

B) Bradycardia Changes in vital signs indicative of increased intracranial pressure are known as Cushing's triad, which consists of increasing systolic pressure with a widening pulse pressure, bradycardia with a full and bounding pulse, and irregular respirations.

You are giving Mr. Pratt his nitro ointment using the paper applicator. You will do all of the following (select all) a. Place new ointment on paper applicator onto the upper arm b. Massage ointment into skin to speed absorption c. Avoid getting ointment on you

B, C

The nurse is providing care for a patient who has been admitted to the hospital with a head injury and who requires regular neurologic vital signs. Which of the following assessments will be components of the patient's score on the Glasgow Coma Scale (GCS) (select all that apply)? A. Judgment B. Eye opening C. Abstract reasoning D. Best verbal response E. Best motor response F. Cranial nerve function

B, D, E The three dimensions of the GCS are eye opening, best verbal response, and best motor response.

The nurse is assessing a patient with angina pectoris. Which of the following are characteristic of the substernal chest pain that occurs with this condition? Select all that apply. a. Occurs without cause b. Radiates to the left arm c. last less than 15 minutes d. usually occurs in the morning e. Is relieved by rest and nitroglycerine f. Is precipitated by exertion or stress

B, E, F

Which of the following dysrhythmias presents on ECG with saw tooth p waves? a. atrial fibrillation b. atrial flutter c. 2nd degree heart block d. complete heart block

B. atrial flutter

An 18-year-old patient was hit in the head with a baseball during practice. When discharging him to the care of his mother, the nurse give which instruction? a. "Watch him for keyhole pupil for the next 24 hours." b. "Expect profuse vomiting for 24 hours after the injury." c. "Wake him every hour and assess his orientation to person time, and place." d. "Notify the physician immediately if he has a headache."

C

Which class of drugs is most widely used in the treatment of cardiomyopathy? Anticoagulants Beta-adrenergic blockers Calcium channel blockers Nitrates

Beta blocker - By decreasing the heart rate & contractility, beta-blockers improve myocardial filling & cardiac output, which are primary goals in the treatment of cardiomyopathy. Antihypertensives aren't usually indicated because they would decrease cardiac output in pts who are already hypotensive. Calcium channel blockers are sometimes used for the same reasons as beta-blockers; however, they aren't as effective as beta-blockers & cause increased hypotension. Nitrates aren't used because of their dilating effects, which would further compromise the myocardium.

A nurse working in a rural county' Public Health Department has been alerted that there is an outbreak of tuberculosis (TB) in the area. The person most at risk for developing TB would be? a. A 16-year-old female high school student b. A 33-year-old day-care worker. c. A 43-year-old homeless man with a history of alcoholism d. A 54-year-old businessman.

C

A patient asks the nurse, "What is the difference between rheumatoid arthritis and osteoarthritis?" What is the most appropriate response by the nurse? a. Osteoarthritis is gender specific; rheumatoid arthritis is not. b. Osteoarthritis is a systemic disease: rheumatoid arthritis is localized. c. Osteoarthritis is a localized disease: rheumatoid arthritis is systemic. d. Osteoarthritis has dislocations ad subluxations; rheumatoid arthritis does not.

C

A patient is diagnosed with uncomplicated rheumatoid arthritis. The nurse explains to the patient that nonsteeroidal anti-inflammatory drugs (NSAIDS) are used in the treatment plan. Which NSAID medication is used to treat rheumatoid arthritis? a. Furosemide b. Haloperidol c. Ibuprofen d. Methotrexate

C

A patient was admitted to the hospital because of a transient ischemic attack secondary to atrial fibrillation. The nurse anticipates the physician will prescribe which medication? a. Digoxin (Lanoxin) b. Diltiazem (Cardizem) c. Warfarin (Coumadin) d. Quinidine gluconate

C

The nurse determines that a patient is at risk for developing anemia if which of the following predisposing factors is identified? a. colostomy following colon resection b. Gastroesophageal reflux disease (GERD) c. Gastrectomy d. Bouts of dumping syndrome

C

The nurse is aware that a patient who has just experienced a myocardial infarction (MI) is most at risk for developing: a. cardiogenic shock. b. heart failure. c. arrhythmias. d. pericarditis.

C

The nurse is aware that a patient with a diagnosis of thrombocytopenia would be likely to complain of which of the following? a. Weakness and fatigue b. Dizziness and vomiting c. Bruising and petechiae d. Light-headedness and nausea

C

The nurse is caring for a patient diagnosed with a fracture. The health care provider ordered a high-protein diet. The nurse explains to the patient that the high-protein diet is ordered for which of the following reasons? a. Protein promotes gluconeogenesis. b. Protein has anti-inflammatory properties. c. Protein promotes cell growth and bone union. d. Protein decreases pain medication requirements

C

The nurse is caring for a patient with low back pain. Which action may the nurse delegate to the nursing assistant? a. Assess pain level. b. Palpate the abdomen for distension. c. Reposition the patient from side-lying to back. d. Assess the patient's skin for skin breakdown.

C

The nurse is developing a plan of care for a patient diagnosed with rheumatoid arthritis. What is the goal of treatment? a. To cure the disease b. To prevent osteoporosis c. To control inflammation d. To encourage bone regeneration

C

The nurse is preparing to assess a patient for jugular vein distention. How should the nurse position the head of the patient's bed? a. High fowler's b. Raised 10 degrees c. Raised 30 degrees d. Supine

C

While assessing a patient with disseminated intravascular coagulation (DIC), the nurse suspects the patient has developed internal bleeding. The nurse should assess the patient for: a. hypertension b. petechiae c. increasing abdominal girth d. bradycardia

C

A patient is admitted to the telemetry floor with acute chest pain radiating down his left arm. The nurse anticipates that which of the following laboratory studies would be ordered to evaluate myocardial damage? Select all that apply. a. Hemoglobin and hematocrit b. Serum glucose c. Creatinine phosphokinase (CPK) d. Troponin T and troponin I e. Myoglobin f. Blood urea nitrogen BUN

C, D, E

A 28-year-old male patient is diagnosed with polycystic kidney disease. Which information is most appropriate for the nurse to include in teaching at this time? A. Complications of renal transplantation B. methods for treating severe chronic pain C. Discussion of options for genetic counseling D. Differences between hemodialysis and peritoneal dialysis

C. Discussion of options for genetic counseling

A 37-year-old female patient is hospitalized with acute kidney injury (AKI). Which information will be most useful to the nurse in evaluating improvement in kidney function? A. Urine volume B. Creatinine level C. Glomerular filtration rate (GFR) D. Blood urea nitrogen (BUN) level

C. Glomerular filtration rate (GFR)

The nurse is caring for a patient with acute renal injury. The nurse should expect hypertonic glucose, insulin infusions, and sodium bicarbonate to be used to treat: A. Hypernatremia B. Hypokalemia C Hyperkalemia D. Hypercalcemia

C. hyperkalemia

A 78-year old patient complains that his food just does not taste like it used to. Which intervention should the medical unit nurse implement first? Ask the dietician to consult with the patient on food preferences. Ask the HCP to suggest an over-the-counter nutritional supplement Medicate the patient before meals with an antiemetic medication Check the patient's current weight with his usual weight.

Check the patient's current weight with his usual weight.

An appropriate nursing intervention for a patient with fatigue related to cancer treatment includes teaching the patient to: Increase fluid intake. Minimize naps or period of rest during the day. Conserve energy by prioritizing activities. Limit dietary intake of high-fiber foods.

Conserve energy by prioritizing activities.

The patient diagnosed with lung cancer has a hemoglobin of 13.4 mg/dL and hematocrit of 40.1; a WBC count of 7800 and a neutrophil count of 62%. Which action should the nurse implement? Place the patient in reverse isolation Notify the HCP Make sure no flowers are taken into the room. Continue to monitor the patient

Continue to monitor the patient

A patient is admitted to the emergency department after complaining of acute chest pain radiating down the left arm. The patient appears anxious, dyspneic, and diaphoretic. Which laboratory studies would the nurse anticipate? Select all that apply. Hemoglobin and hematocrit (HCT) Serum glucose Creatine kinase (CK) Troponin T and Troponin I Myoglobin Blood urea nitrogen (BUN)

Creatinine troponin Myoglobin

A patient has undergone an amputation of three toes and a femoral-popliteal bypass. The nurse should teach the patient that after surgery which of the following leg positions is contraindicated while sitting in a chair? Crossing the legs. Elevating the legs. Flexing the ankles. Extending the knees.

Crossing the legs

A nurse is assessing a bedridden patient and notes sacral edema. The nurse determines that the edema is most likely the rsult of which of the following? a. Diabetes mellitus b. Pulmonary emboli c. Chronic kidney disease d. Right-sided heart failure

D

A patient is diagnosed with fat emboli. Which signs and symptoms would the nurse expect to find during assessment? a. Tachypnea, tachycardia, shortness of breath and paresthesia b. Paresthesia, bradypnea, bradycardia, and petechial rash on chest and neck c. Bradypena, bradycardia, shortness of breath, and petechial rash on chest and neck d. Tachypnea, tachycardia, shortness of breath, and petechial rash on chest and neck

D

A patient with a myocardial infarction asks the nurse why he is receiving morphine. What is the best response by the nurse? a. To sedate the patient b. To decrease the patient's pain c. To decrease the patient anxiety d. To decrease oxygen demand on the patient's heart

D

The nurse is reviewing patient's charts. Select the patient who would be most at risk for developing anemia. a. a 2-year old in day care b. A 22-year-old college student c. A 55-year-old neighbor d. An elderly nursing home resident

D

The nurse is teaching patients about interventions to increase survival rates of patients with lung cancer. The nurse determines teaching is effective when the patient states that which action will increase survival? a. Early bronchoscopy b. Early detection c. High-dose chemotherapy d. Smoking cessation

D

The nurse performs an assessment on a newly admitted patient. The data include dyspnea, cough, weight gain, weakness, and edema. The nurse interprets these as signs and symptoms of: a. pericarditis. b. hypertension. c. myocardial infarction (MI). d. heart failure.

D

What is the most important information for a nurse to teach a patient with chronic obstructive pulmonary disease? a. How to assess his own pulse and respiratory rates. b. How to recognize when a change is needed in in his oxygen therapy c . How to treat respiratory infections without use of antibiotics. D. How to recognize the signs of an impending respiratory infection.

D

Which condition is most closely associated with weight gain, nausea, and a decrease in urine output? a. angina pectoris b. Cardiomyopathy c. Left-sided heart failure d. Right-sided heart failure

D Weight gain, nausea, & a decrease in urine output are secondary effects of right-sided heart failure. Cardiomyopathy is usually identified as a symptom of left-sided heart failure. Left-sided heart failure causes primarily pulmonary symptoms rather than systemic ones. Angina pectoris doesn't cause weight gain, nausea, or a decrease in urine output.

The nurse assesses a patient for signs of meningeal irritation and observes her for nuchal rigidity. Which of the following indicates the presence of this sign of meningeal irritation? Tonic spasms of the legs Curling in a fetal position Arching of the neck and back Resistance to flexion of the neck

D) Resistance of flexion of the neck Nuchal rigidity is a clinical manifestation of meningitis. During assessment, the patient will resist passive flexion of the neck by the health care provider.

The nurse determines that instruction regarding prevention of future urinary tract infections (UTIs) has been effective for a 22-year-old female patient with cystitis when the patient states which of the following? A. "I can use vaginal antiseptic sprays to reduce bacteria." B. "I will drink a quart of water or other fluids every day." C. "I will wash with soap and water before sexual intercourse." D. "I will empty my bladder every 3 to 4 hours during the day."

D. "I will empty my bladder every 3 to 4 hours during the day."

A patient with end-stage renal disease (ESRD) is scheduled for his first hemodialysis treatment. The patient asks the nurse what common complications may occur from the treatment. What would be the nurse's best reply? A. "High blood sugar levels and low protein levels may occur." B. "Bleeding and double vision may occur." C. "Confusion and diarrhea may occur." D. "Low blood pressure and cramping sometimes occur."

D. "Low blood pressure and cramping sometimes occur."

Which assessment data reported by a 28-year-old male patient is consistent with a lower urinary tract infection (UTI)? A. Poor urine output B. Bilateral flank pain C. Nausea and vomiting D. Burning on urination

D. Burning on urination

Prior to administering medication through a NG tube, it is important to: a. Crush only EC and SR tablets b. Flush with 30 Cc of sterile water before instilling medication c. Instill medication quickly using 75cc syringe d. Chest for placement of NG tube

D. Chest for placement of NG tube

Which characteristics are associated with urge incontinence (select all that apply)? A. Treated with Kegel exercises B. Found following prostatectomy C. Common in postmenopausal women D. Involuntary urination preceded by urgency E. Caused by overactivity of the detrusor muscle F. Bladder contracts by reflex, overriding central inhibition

D. Involuntary urination preceded by urgency E. Caused by overactivity of the detrusor muscle F. Bladder contracts by reflex, overriding central inhibition

A 68-year-old female patient admitted to the hospital with dehydration is confused and incontinent of urine. Which nursing action will be best to include in the plan of care? A. Restrict fluids between meals and after the evening meal. B. Apply absorbent incontinent pads liberally over the bed linens. C. Insert an indwelling catheter until the symptoms have resolved. D. assist the patient to the bathroom every 2 hours during the day.

D. assist the patient to the bathroom every 2 hours during the day.

A 32-year-old who is employed as a hairdresser and has a 15 pack-year history of cigarette smoking is scheduled for an annual physical examination. The nurse will plan to teach the patient about the increased risk for A. renal failure. B. kidney stones. C. pyelonephritis D. bladder cancer

D. bladder cancer

The nurse teaches a 64-year-old woman to prevent the recurrence of renal calculi by A. using a filter to strain all urine. B. avoiding dietary sources of calcium. C. choosing diuretic fluids such as coffee. D. drinking 2000 to 3000 ml of fluid a day.

D. drinking 2000 to 3000 ml of fluid a day.

When assessing motor function of a patient admitted with a stroke, the nurse notes mild weakness of the arm demonstrated by downward drifting of the arm. The nurse would most accurately document this finding as a. athetosis b. hypotonia c. hemiparesis d. pronator drift

D. pronator drift

The nurse is working with a 73-year-old patient with osteoarthritis (OA). In assessing the patient's understanding of this disorder, the nurse concludes teaching has been effective when the patient describes the condition as which of the following? Joint destruction caused by an autoimmune process Degeneration of articular cartilage in synovial joints Overproduction of synovial fluid resulting in joint destruction Breakdown of tissue in non-weight-bearing joints by enzymes

Degeneration of articular cartilage in synovial joints

In the patient experiencing ventricular fibrillation (VF), what is the rationale for using cardiac defibrillation? Enhance repolarization and relaxation of ventricular myocardial cells Provide an electrical impulse that stimulates normal myocardial contractions Depolarize the cells of the myocardium to allow the SA node to resume pacemaker function Deliver an electrical impulse to the heart at the time of ventricular contraction to convert the heart to a sinus rhythm

Depolarize the cells of the myocardium to allow the SA node to resume pacemaker function

A patient with a hemoglobin (Hgb) level of 7.8 g/dL has cardiac palpitations, a heart rate of 102 bpm, and an increased reticulocyte count. At this severity of anemia, what other manifestations would the nurse expect the patient to exhibit? Pallor Dyspnea A smooth tongue Sensitivity to cold

Dyspnea Reticulocyte: Immature RBC w/o nucleus rational Hgb level indicates a moderate anemia additional findings usually include dyspnea and fatigue

An adult with a history of chronic obstructive pulmonary disease (COPD)and metastatic carcinoma of the lung has not responded to radiation therapy and is being admitted to the hospice program. The nurse should conduct a fucused patient assessment for: Ascites. Pleural friction rub. Dyspnea. Peripheral edema.

Dyspnea

Cancer prevalence is defined as: The likelihood cancer will occur in a lifetime. The number of persons with cancer at a given point in time. The number of new cancers in a year. All cancer cases more than 5 years old.

The number of persons with cancer at a given point in time.

The nurse is caring for a patient who has osteoarthritis (OA) of the knees. The nurse teaches the patient that the most beneficial measure to protect the joints is to do which of the following? Use a wheelchair to avoid walking as much as possible. Eat a well-balanced diet to maintain a healthy body weight. Use a walker for ambulation to relieve the pressure on her hips. Sit in chairs that do not cause her hips to be lower than her knees.

Eat a well-balanced diet to maintain a healthy body weight.

The nurse is teaching the patient who is receiving chemotherapy and the family how to manage possible nausea and vomiting at home. The nurse should include information about: Eating frequent, small meals throughout the day. Eating three normal meals a day. Eating only cold foods with no odor. Limiting the amount of fluid intake.

Eating frequent, small meals throughout the day.

The nurse evaluates that the patient correctly understands how to report signs and symptoms of bleeding when the patient makes which of the following statements? "Petechiae are large, red skin bruises." "Ecchymoses are large, purple skin bruises." "Purpura is an open cut on the skin." "Abrasions are small pinpoint red dots on the skin."

Ecchymoses are large, purple skin bruises

The nurse formulates a nursing diagnosis of impaired physical mobility related to decreased muscle strength for a 78-year-old patient following left total knee replacement. Which of the following would be an appropriate nursing intervention for this patient? Promote vitamin D and calcium intake in the diet. Provide passive range of motion to all of the joints q4hr. Encourage isometric quadriceps-setting exercises at least qid. Keep the left leg in extension and abduction to prevent contractures.

Encourage isometric quadriceps-setting exercises at least qid.

A patient with hemophilia comes to the clinic for treatment. What should the nurse anticipate that he or she will need to administer? Whole blood Thromboplastin Factor concentrates Fresh frozen plasma

Factor concentrates Although whole blood and fresh frozen plasma contain the clotting factors that are deficient in hemophilia, specific factor concentrates have been developed that are more pure and safer in preventing infection transmission. Thromboplastin is factor III and is not deficient in patients with hemophilia.

Which of the following is the most reliable early indicator of infection in a patient who is neutropenic? Fever Chills Tachycardia Dyspnea

Fever

Which of the following is the most reliable early indicator of infection in a patient who is neutropenic? Fever. Chills. Tachycardia. Dyspnea.

Fever

The staff nurse is caring for a patient who was diagnosed with pancreatic cancer during exploratory surgery. Which problem is the priority for postoperative Day One? Ineffective coping Fluid and electrolyte imbalance Risk for infection Potential for suicidal thoughts

Fluid and electrolyte imbalance

What is the most important method for identifying the presence of infection in a neutropenic patient? Frequent temperature monitoring Routine blood and sputum cultures Assessing for redness and swelling Monitoring white blood cell (WBC) count

Frequent temperature monitoring (Rationale- An elevated temperature is of most significance in recognizing the presence of an infection in the neutropenic patient because there is no leukocytic response, and when the WBC count is depressed, the normal phagocytic mechanisms of infection are impaired, and the class signs of inflammation may not occur. Cultures are indicated if temperature is elevated but are not use to monitor for infection.)

The nurse is assessing an older Caucasian male who has a history of peripheral vascular disease. The nurse observes that the man's left great toe is black. The discoloration is probably a result of: Atrophy Contraction Gangrene Rubor

Gangrene

Assessment of a patient taking a nonsteroidal anti-inflammatory drug (NSAID) for pain management should include specific questions regarding which of the following systems? Gastrointestinal. Renal. Pulmonary. Cardiac.

Gastrointestinal

A patient with acute asthma is prescribed short-term corticosteroid therapy. Which is the expected outcome for the use of steroids in patients with asthma? Promote bronchodilation Act as an expectorant. Have an anti-inflammatory effect. Prevent development of respiratory infections.

Have an anti-inflammatory effect

Which of the following signs and symptoms in a patient with a T4 spinal cord injury should alert the nurse to the possibility of autonomic dysreflexia? Headache and rising blood pressure Irregular respirations and shortness of breath Decreased level of consciousness or hallucinations Abdominal distention and absence of bowel sounds

Headache and rising blood pressure Manifestations of autonomic dysreflexia are hypertension (up to 300 mm Hg systolic), a throbbing headache, bradycardia, and diaphoresis. Respiratory manifestations, decreased level of consciousness, and gastrointestinal manifestations are not characteristic manifestations.

A nursing educator is speaking with a group of new graduates about a patient who has acute glomerulonephritis. The educator should tell the nurses that the patient may exhibit which of the following clinical manifestations? A. Hematuria B. Decrease in serum creatinine levels C. Hypotension D. Glucosuria

Hematuria

The nurse is planning care for a 68-year-old patient with an abdominal mass and suspected bowel obstruction. Which of the following factors in the patient's history increases the patient's risk for colorectal cancer? Osteoarthritis History of rectal polyps History of lactose intolerance Use of herbs as dietary supplements

History of rectal polyps

The nurse is assessing a 60-year old who has hoarseness. The nurse should conduct a focused assessment to determine: Patterns of medication use and history of alcohol consumption Exposure to sun and family history of head and neck cancers Exposure to wood dust and high fat diet History of tobacco and alcohol use

History of tobacco and alcohol use

The nurse is teaching a patient about the risk factors of osteoporosis. It is most important for the nurse to include which factors? Select all that apply. Inadequate dietary intake of calcium Blood pressure medications Family history Smoking Oral hypoglycemics

Inadequate dietary intake of calcium Family history Smoking

Which of the following is an assessment finding associated with internal bleeding with disseminated intravascular coagulation? Bradycardia. Hypertension. Increasing abdominal girth. Petechiae.

Increasing abdominal girth

When planning care for a patient with a C5 spinal cord injury, which nursing diagnosis is the highest priority? Risk for impairment of tissue integrity caused by paralysis Altered patterns of urinary elimination caused by quadriplegia Altered family and individual coping caused by the extent of trauma Ineffective airway clearance caused by high cervical spinal cord injury

Ineffective airway clearance caused by high cervical spinal cord injury Maintaining a patent airway is the most important goal for a patient with a high cervical fracture. Although all of these are appropriate nursing diagnoses for a patient with a spinal cord injury, respiratory needs are always the highest priority. Remember the ABCs.

When planning care for a patient with cirrhosis, the nurse will give highest priority to which of the following nursing diagnoses? • Imbalanced nutrition: less than body requirements • Impaired skin integrity related to edema, ascites, and pruritus • Excess fluid volume related to portal hypertension and hyperaldosteronism • Ineffective breathing pattern related to pressure on diaphragm and reduced lung volume

Ineffective breathing pattern related to pressure on diaphragm and reduced lung volume

In which areas of the United States is the incidence of tuberculosis highest? Rural farming areas. Inner-city areas. Areas where clean water standards are low. Suburban areas with significant industrial pollution.

Inner city areas

Which of the following drug classes should be administered to a patient with heart failure to maximize cardiac performance? Beta-adrenergic blockers Calcium channel blockers Diuretics Inotropic agents

Inotropic agents

In teaching the patient with pernicious anemia about the disease, the nurse explains that it results from a lack of folic acid intrinsic factor extrinsic factor cobalamin intake

Intrinsic factor rationale: Pernicious anemia is a vitamin B 12 deficiency that results when parietal cells in the stomach fail to secrete enough intrinsic factor to absorb ingested cobalamin. Extrinsic factor is cobalamin.

Which of the following mental status changes may occur when a patient with pneumonia is first experiencing hypoxia? Coma. Apathy. Irritability. Depression.

Irritability

The nurse is caring for a patient with osteoarthritis who is about to undergo total left knee arthroplasty. The nurse assesses the patient carefully to be sure that there is no evidence of which of the following in the preoperative period? Pain Left knee stiffness Left knee infection Left knee instability

Left knee infection

While assessing a thoracotomy incisional area from which a chest tube exits, the nurse feels a crackling sensation under the fingertips along the entire incision. Which of the following should be the nurse's first action? Lower the head of the bed and call the physician. Prepare an aspiration tray. Mark the area with a skin pencil at the outer periphery of the crackling. Turn off the suction of the chest drainage system.

Mark the area with a skin pencil at the outer periphery of the crackling

Which assessment finding in a patient with acute pancreatitis would the nurse need to report most quickly to the health care provider? • Nausea and vomiting • Hypotonic bowel sounds • Abdominal tenderness and guarding • Muscle twitching and finger numbness

Muscle twitching and finger numbness

The nurse is performing a musculoskeletal assessment of an 81-year-old female patient whose mobility has been progressively decreasing in recent months. How should the nurse best assess the patient's range of motion (ROM) in the affected leg? Perform passive ROM, asking the patient to report any pain. Ask the patient to lift progressive weights with the affected leg. Observe the patient's unassisted ROM in the affected leg. Move both of the patient's legs from a supine position to full flexion.

Observe the patient's unassisted ROM in the affected leg.

A nurse is caring for a patient who is 2 days post left total hip replacement and complaining of severe pain in the left leg. The nurse administers the prescribed morphine sulfate 2mg i>V. The patient continues to complain of severe pain. The nurse assesses the patient's left leg and finds the extremity cool to touch with absent pulses and a capillary refill greater than 3 seconds. What is the priority action of the nurse? Notify the health care provider Document the clinical findings. Readminister the prescribed morphine sulfate. Reassess the left lower extremity within 1 hour.

Notify the health care provider

The nurse is reinforcing general health teaching with a 64-year-old patient with osteoarthritis (OA) of the hip. Which of the following points would the nurse include in this review of the disorder (select all that apply)? OA cannot be successfully treated with any current therapy options. OA is an inflammatory disease of the joints that may present symptoms at any age. Joint degeneration with pain and disability occurs in the majority of people by the age of 60. OA is more common with aging, but usually it remains confined to a few joints and does not cause crippling. OA can be prevented from progressing when well controlled with a regimen of exercise, diet, and medication.

OA is more common with aging, but usually it remains confined to a few joints and does not cause crippling. OA can be prevented from progressing when well controlled with a regimen of exercise, diet, and medication.

A patient who has had several episodes of bloody diarrhea is admitted to the emergency department. Which action should the nurse anticipate taking? Obtain a stool specimen for culture. Administer antidiarrheal medications. Teach about adverse effects of nonsteroidal anti-inflammatory drugs (NSAIDs). Provide education about antibiotic therapy.

Obtain a stool specimen for culture.

During the assessment of a patient with cobalamin deficiency, what manifestations would the nurse expect to find in the patient? Icteric sclera Hepatomegaly Paresthesia of the hands and feet Intermittent heartburn with acid reflux

Paresthesia of the hands and feet Icteric sclera (jaundice, yellow ) Paresthesia of the hands and feet. rationale: weakness, paresthesias, impaired thought process are characteristic of vitamin B 12 deficiency.

A 67-year-old patient hospitalized with osteomyelitis has an order for bed rest with bathroom privileges with the affected foot elevated on two pillows. The nurse would place highest priority on which of the following interventions? Ambulate the patient to the bathroom every 2 hours. Ask the patient about preferred activities to relieve boredom. Allow the patient to dangle legs at the bedside every 2 to 4 hours. Perform frequent position changes and range-of-motion exercises.

Perform frequent position changes and range-of-motion exercises.

Which of the following nursing interventions is most appropriate when turning a patient following spinal surgery? Placing a pillow between the patient's legs and turning the body as a unit. Having the patient turn to the side by grasping the side rails to help turn over. Elevating the head of bed 30 degrees and having the patient extend the legs while turning. Turning the patient's head and shoulders and then the hips, keeping the patient's body centered in the bed.

Placing a pillow between the patient's legs and turning the body as a unit.

The nurse is caring for a patient diagnosed with a fracture. The health care provider ordered a high-protein diet. The nurse explains to the patient that the high-protein diet is ordered for which of the following reasons? a. Protein promotes gluconeogenesis. Protein has anti-inflammatory properties. Protein promotes cell growth and bone union. Protein decreases pain medication requirements.

Protein promotes cell growth and bone union.

The nurse is caring for a 76-year-old man who has undergone left knee arthroplasty with prosthetic replacement of the knee joint to relieve the pain of severe osteoarthritis. Postoperatively the nurse expects which of the following will be included in the care of the affected leg? Progressive leg exercises to obtain 90-degree flexion Early ambulation with full weight bearing on the left leg Bed rest for 3 days with the left leg immobilized in extension Immobilization of the left knee in 30-degree flexion for 2 weeks to prevent dislocation

Progressive leg exercises to obtain 90-degree flexion

Which finding by the nurse for a patient with a nursing diagnosis of impaired gas exchange will be most useful in evaluating the effectiveness of treatment? Even, unlabored respirations Pulse oximetry reading of 92% Respiratory rate of 18 breaths/minute Absence of wheezes, rhonchi, or crackles

Pulse ox reading of 92

The nurse is caring for a patient with low back pain. Which action may the nurse delegate to the nursing assistant? Assess pain level. Palpate the abdomen for distension. Reposition the patient from side-lying to back. Assess the patient's skin for skin breakdown.

Reposition the patient from side-lying to back.

In the normal heart, the pacemaker is A. av node b. sinoatrial node c. purkinje fibers d. left bundle branch

Sinoatrial node

The nurse is caring for a patient admitted with a spinal cord injury following a motor vehicle accident. The patient exhibits a complete loss of motor, sensory, and reflex activity below the injury level. The nurse recognizes this condition as which of the following? Central cord syndrome Spinal shock syndrome Anterior cord syndrome Brown-Séquard syndrome

Spinal shock syndrome About 50% of people with acute spinal cord injury experience a temporary loss of reflexes, sensation, and motor activity that is known as spinal shock. Central cord syndrome is manifested by motor and sensory loss greater in the upper extremities than the lower extremities. Anterior cord syndrome results in motor and sensory loss but not reflexes. Brown-Séquard syndrome is characterized by ipsilateral loss of motor function and contralateral loss of sensory function.

What is a major method of preventing infection in the patient with neutropenia? Prophylactic antibiotics A diet that eliminates fresh fruits and vegetables High-efficiency particulate air (HEPA) filtration Strict hand washing by all persons in contact with the patient

Strict hand washing by all persons in tact with the patient

Which early symptom does the nurse observe in a patient with thrombocytopenia who has developed a hemorrhage? Tachycardia Bradycardia Decreased PaCo2 Narrow pulse pressure

Tachycardia The nurse would observe tachycardia in the hemorrhaging client because the heart beats faster to compensate for the decreased circulating volume and decreased number of RBC.

Which assessment finding in a patient who had a total gastrectomy 12 hours previously is most important to report to the health care provider? Absent bowel sounds Scant nasogastric (NG) drainage. Complaints of incisional pain Temperature 102.1 F (38.9C)

Temperature 102.1 F (38.9C)

The nurse is caring for patients on an oncology unit and is administering medications. Which medication should the nurse administer first? The anti- nausea medication for the patient who is nauseated and thinks he may vomit The pain medication to the female patient who has 2/10 pain The loop diuretic to the female patient who had an output greater than intake The nitroglycerin paste to the male patient who is diagnosed with angina pectoris

The anti- nausea medication for the patient who is nauseated and thinks he may vomit

A patient who was admitted with acute bleeding from esophageal varices asks the nurse the purpose for the ordered ranitidine (Zantac). Which response by the nurse is most appropriate? • The medication will reduce the risk for aspiration. • The medication will decrease nausea and anorexia. • The medication will inhibit the development of gastric ulcers. • The medication will prevent irritation to the esophageal varices.

The medication will prevent irritation to the esophageal varicies

Which of the following patients is at highest risk for colorectal cancer? The patient who smokes The patient who eats a vegetarian diet. The patient who has been treated for Crohn's disease for 20 years The patient who has a family history of lung cancer

The patient who has been treated for Crohn's disease for 20 years

A patient is in the emergency department with a suspected fracture of the right hip. Which assessment(s) should the nurse expect to find? Select all that apply. The right leg is longer than the left leg. The right leg is shorter than the left leg. The right leg is externally rotated. The right leg is internally rotated. The right leg is abducted. The right leg is adducted.

The right leg is shorter than the left leg. The right leg is externally rotated. The right leg is adducted.

Which of the following characteristics of a patient's recent seizure is congruent with a partial seizure? • The patient lost consciousness during the seizure. • The seizure involved lip smacking and repetitive movements. • The patient fell to the ground and became stiff for 20 seconds. • The etiology of the seizure involved both sides of the patient's brain.

The seizure involved lip smacking and repetitive movements.

Which of the following is an expected outcome of pursed-lip breathing for patients with emphysema? To promote oxygen intake. To strengthen the diaphragm. To strengthen the intercostal muscles To promote carbon dioxide elimination

To promote CO2 elimination

Ectopy is caused by an irritability in the heart muscle T/F?

True

It is normal to see sinus bradycardia in athletes during sleep T/F?

True

Which of the following interventions should the nurse perform in the acute care of a patient with autonomic dysreflexia? Urinary catheterization Administration of benzodiazepines Suctioning of the patient's upper airway Placement of the patient in the Trendelenburg position

Urinary catheterization Because the most common cause of autonomic dysreflexia is bladder irritation, immediate catheterization to relieve bladder distention may be necessary. Benzodiazepines are contraindicated, and suctioning is likely unnecessary. The patient should be positioned upright.

Which nursing action will be included in the plan of care for a patient with cirrhosis who has ascites and 4+ edema of the feet and legs? • Restrict dietary protein intake. • Reposition the patient every 4 hours. • Use a pressure-relieving mattress. • Perform passive range of motion qid.

Use a pressure reliving mattress

The nurse is discharging a patient admitted with a transient ischemic attack (TIA). For which of the following medications might the nurse expect to provide discharge instructions (select all that apply)? a. clopidogrel (plavix) b. enoxaparin (lovenox) c. enteric coated aspirin (ecotrin) d. tissue plasminogen activated (tPA)

a. clopidogrel (plavix) c. enteric coated aspirin (ecotrin)

A patient with sickle cell anemia asks the nurse why the sickling crisis does not stop when oxygen therapy is started. Which explanation should the nurse give to the patient? Sickling occurs in response to decreased blood viscosity, which is not affected by oxygen therapy When RBCs sickle, they occlude small vessels, which cause more local hypoxia and more sickling. The primary problem during a sickle cell crisis is destruction of the abnormal cells, resulting in fewer RBCs to carry oxygen. Oxygen therapy does not alter the shape of the abnormal erythrocytes but only allows for increased oxygen concentration in hemoglobin.

When RBCs sickle, they occlude small vessels, which cause more local hypoxia and more sickling. During a sickle cell crisis, the sickling cells clog small capillaries and the resulting hemostasis promotes a self- perpetuating cycle of local hypoxia, deoxygenation of more erythrocytes, and more sickling. Administration of oxygen may help to control further sickling but additional oxygen does not reach areas of local hypoxia caused by occluded vessels.

A patient is hospitalized with vomiting of "coffee- ground" emesis. The nurse will anticipate preparing the patient for endoscopy. angiography. gastric analysis testing. barium contrast studies.

endoscopy

A patient is brought to the emergency department with a knife impaled in the abdomen following a domestic fight. During the initial assessment of the patient, the nurse should assess the BP and pulse. remove the knife to assess the wound. determine the presence of Rovsing sign. insert a urinary catheter and assess for hematuria.

assess the BP and pulse.

While assessing a patient with dilated cardiomyopathy, the nurse notices that the electrocardiogram (ECG) rhythm no longer has any P waves, only a fine wavy line. The ventricular rhythm is irregular with a QRS duration of 0.08 seconds. The heart rate is 110 beats/minute. The nurse interprets this rhythm as: atrial fibrillation. ventricular fibrillation. atrial flutter. sinus tachycardia.

atrial fibrillation

A patient with pericarditis has been admitted to the CCU. The nurse caring for the patient knows that the most common clinical manifestation of pericarditis is what? a. Palpitations b. Chest pain c. Burning sensation in chest d. Pounding heart rate

b. Chest pain

When providing community health care teaching regarding the early warning signs of Alzheimer's disease, which of the following signs would the nurse advise family members to report (select all that apply)? A. Misplacing car keys B. Losing sense of time C. Difficulty performing familiar tasks D. Problems with performing basic calculations E. Becoming lost in a usually familiar environment

b. Losing sense of time c. Difficulty performing familiar tasks d. Problems with performing basic calculations e. Becoming lost in a usually familiar environment

A patient with mitral stenosis exhibits symptoms of a dysrhythmia. Based on the pathophysiology of the disease process, the nurse would expect the patient to exhibit what heart rhythm? a. Normal sinus rhythm b. Supraventricular tachycardia c. Atrial fibrillation d. Electromechanical dissociation

c. Atrial fibrillation

Which topic will the nurse plan to include in discharge teaching for a patient with systolic health failure and ejection fraction of 33% a. Need to begin an aerobic exercise program several times weekly b. Use of salt substitutions to replace table salt when cooking and at the table c. Benefits and side effects of angiotensin-converting enzyme (ACE) inhibitors d. Importance of making an annual appointment with the primary care provider

c. Benefits and side effects of angiotensin-converting enzyme (ACE) inhibitors

Which of the following nursing interventions is most appropriate when communicating with a patient suffering from aphasia post stroke? a. present several thoughts at once so that the patient can connect the ideas b. ask open ended questions to provide the patient with opportunity to speak c. use simple, short sentences accompanied by visual cues to enhance comprehension d. finish the patient's sentences so as to minimize frustration associated with slow speech

c. use simple, short sentences accompanied by visual cues to enhance comprehension

Which of the following nursing interventions is most appropriate when caring for patients with dementia? • Avoid direct eye contact. • Lovingly call the patient "honey" or "sweetie." • Give simple directions, focusing on one thing at a time. • Treat the patient according to his or her age-related behavior.

• Give simple directions, focusing on one thing at a time.

A patient with Crohn's disease has megaloblastic anemia. The nurse will anticipate teaching the patient about the ongoing need for oral ferrous sulfate tablets regular blood transfusions. iron dextran (Imferon) infusion. cobalamin (B12) nasal spray or injections.

cobalamin (B12) nasal spray or injections. Vitamin B-12 and folic acid deficiencies and certain medications are the most common causes of megaloblastic anemia,

Which of the following signs and symptoms in a patient with a T4 spinal cord injury should alert the nurse to the possibility of autonomic dysreflexia? • Headache and rising blood pressure • Irregular respirations and shortness of breath • Decreased level of consciousness or hallucinations • Abdominal distention and absence of bowel sounds

• Headache and rising blood pressure

Your patient requests his water pill but when you check the medical record no diuretic has been ordered. When you explain this to your patient, he stated, that's okay. I have my own here I can take. Your first response is: a. Carefully check dosage and prescription on the bottle before allowing patient to take his medication. Then chart fully in the medication record b. Check with the pharmacy or a reference book regarding correct dosage and medication before allowing the patient to take his pill c. Explain to the patient why he cannot take his own pills and report his behaviors to the nurse manager d. Ask the patient not to take the medication as his medication may have changed during his illness. Tell him you will page the PD to double check whether this medication is still appropriate

d. Ask the patient not to take the medication as his medication may have changed during his illness. Tell him you will page the PD to double check whether this medication is still appropriate

Which of the following lab values should the nurse report to the health care provider when the client has anemia? Schilling test result, elevated. Intrinsic factor, absent. Sedimentation rate, 16 mm/h. Red blood cells (RBCs) within normal range.

intrinsic factor, absent

A patient has developed compartment syndrome following application of a cast from a fractured tibia. The nurse is aware that the priority goal of intervention is to: prevent tissue death, which can occur within 2 to 4 hours. decrease the swelling in the extremity. prevent further complications decrease the level of pain.

prevent tissue death, which can occur within 2 to 4 hours.

When planning care for a patient with a C5 spinal cord injury, which nursing diagnosis is the highest priority? • Risk for impairment of tissue integrity caused by paralysis • Altered patterns of urinary elimination caused by quadriplegia • Altered family and individual coping caused by the extent of trauma • Ineffective airway clearance caused by high cervical spinal cord injury

• Ineffective airway clearance caused by high cervical spinal cord injury

A 70-year-old male patient is admitted to the medical-surgical unit with a fractured femur. The patient is placed in traction. The patient asks the nurse to help him with back care. The most appropriate intervention by the nurse is: telling the patient that he can't have back care while he's in traction. telling the patient to use the trapeze to life his back off the bed. supporting the weight to give the patient more slack to move. removing the weight to give the patient more slack to move.

telling the patient to use the trapeze to life his back off the bed.

When a patient is diagnosed with acute hepatitis B, the nurse will plan to teach the patient about • ways to increase exercise and activity level. • self-administration of á-interferon (Intron A). • side effects of nucleoside and nucleotide analogs. • measures that will be helpful in improving appetite.

measures that will be helpful in improving appetitie

Benzodiazepines are indicated in the treatment of cases of delirium that have which of the following causes? • Polypharmacy • Cerebral hypoxia • Alcohol withdrawal • Electrolyte imbalances

• Alcohol withdrawal

The patient's laboratory results show a marked decrease in RBCs, WBCs, and platelets. What term should the nurse use when reporting the results to the physician? Hemolysis Leukopenia Pancytopenia Thrombocytosis

pancytopenia

During physical assessment of a patient with thrombocytopenia what would the nurse expect to find? Sternal tenderness Petechiae and purpura Jaundiced sclera and skin Tender, enlarged lymph nodes

petechia and purport Rational: Petechiae are small, flat, red or reddish brown pinpoint microhemorrhages that occur on the skin when platelet levels are low; when they are numerous, they group, causing reddish bruises known as purpura. Jaundice occurs in accumulation of bile pigments from red blood cells (RBCs). Enlarged, tender lymph nodes are associated with infection, and sternal tenderness is associated with leukemias.

Which of the following are significant data to gather from a client who has been diagnosed with pneumonia? Select all that apply. Quality of breath sounds. Presence of bowel sounds. Occurrence of chest pain. Amount of peripheral edema. Color of nail beds.

quality of breath sounds occurrence of chest pain color of nail beds

Which of the following patients may face the greatest risk of developing delirium? • A patient with fibromyalgia whose chronic pain has recently worsened • An elderly patient whose recent computed tomography shows brain atrophy • A patient with a fracture who has spent the night in the emergency department • An elderly patient who takes multiple medications for various health problems

• An elderly patient who takes multiple medications for various health problems

A nurse should interpret which of the following as an early sign of a tension pneumothorax in a patient with chest trauma? Diminished bilateral breath sounds. Muffled heart sounds. Respiratory distress. Tracheal deviation.

respiratory distress

Which of the following conditions can place a patient at risk for acute respiratory distress syndrome (ARDS)? Septic shock. Chronic obstructive pulmonary disease. Asthma Heart failure.

septic shock

Which of the following clinical manifestations would the nurse interpret as representing neurogenic shock in a patient with acute spinal cord injury? • Bradycardia • Hypertension • Neurogenic spasticity • Bounding pedal pulses

• Bradycardia

The nurse who has administered a dose of risperidone (Risperdal) to a patient with delirium assesses for which of the following intended effects of the medication? • Lying quietly in bed • Alleviation of depression • Reduction in blood pressure • Disappearance of confusion

• Lying quietly in bed

When counseling a patient with a family history of stomach cancer about ways to decrease risk for developing stomach cancer, the nurse will teach the patient to avoid smoked foods such as bacon and ham. foods that cause abdominal distention. chronic use of H2 blocking medications. emotionally or physically stressful situations.

smoked foods such as bacon and ham.

A patient with bacterial pneumonia is to be started on IV antibiotics. Which of the following diagnostic tests must be completed before antibiotic therapy begins? Urinalysis. Sputum culture. Chest radiograph. Red blood cell count.

sputum culture

Which of the following statements by the wife of a patient with Alzheimer's disease (AD) demonstrates an accurate understanding of her husband's medication regimen? • "I'm really hoping his medications will slow down his mental losses." • "We're both holding out hope that this medication will cure his disease." • "I know that this won't cure him, but we learned that it might prevent a bodily decline while he declines mentally." • "I learned that if we are vigilant about his medication schedule, he may not experience the physical effects of his disease."

• "I'm really hoping his medications will slow down his mental losses."

For which of the following patients should the nurse prioritize an assessment for depression? • A patient in the early stages of Alzheimer's disease • A patient who is in the final stages of Alzheimer's disease • A patient experiencing delirium secondary to dehydration • A patient who has become delirious following an atypical drug response

• A patient in the early stages of Alzheimer's disease

Which of the following nursing diagnoses is likely to be a priority in the care of a patient with myasthenia gravis (MG)? • Acute confusion • Bowel incontinence • Activity intolerance • Disturbed sleep pattern

• Activity intolerance

A nurse is caring for a patient with cardiomyopathy and is aware that the patient is at high risk for developing: Heart failure Diabetes mellitus Myocardial infarction Pericardial effusion

Diabetes mellitus ??? I think this is wrong... HF!

Which condition is most closely associated with weight gain, nausea, and a decrease in urine output? Angina pectoris Cardiomyopathy Left-sided heart failure Right-sided heart failure

Right sided heart failure

A patient with peripheral vascular disease returns to the surgical care unit after having femoral popliteal bypass grafting. Indicate in which order the nurse should conduct assessment of this patient. (a) Postoperative pain (b) Peripheral pulses (c) Urine output (d) Incision site a, c, b, d a, d, b, c b, d, c, a d, b, c, a

b, d, c, a

The nurse should assess the patient with left-sided heart failure for which of the following? Select all that apply. Dyspnea. Jugular vein distention (JVD). Crackles. Right upper quadrant pain. Oliguria. Decreased oxygen saturation levels.

Dyspnea Crackles Oliguria Decreased O2 sat

The cyanosis that accompanies bacterial pneumonia is primarily caused by which of the following? Decreased cardiac output. Pleural effusion. Inadequate peripheral circulation Decreased oxygenation of the blood.

Decreased oxygenation of the blood

While caring for a patient with respiratory disease, the nurse observes that the patient's SpO2 drops from 93% to 88% while the patient is ambulating in the hallway. What is the priority action of the nurse? Notify the health care provider. Document the response to exercise. Administer the PRN supplemental O2. Encourage the patient to pace activity.

administer the PRN supplemental O2

You are preparing to interpret an ECG rhythm strip. Place the following steps for ECG rhythm analysis from first to last, in chronological order. Use all of the options. A. Measure the QRS duration B. Interpret the rhythm C. Analyze the P waves D. Determine the rate and rhythm E. Measure the P-R interval

D, C, E, A, B Determine rate and rhythm Analyze the P wave Measure the PR interval measure the QRS duration Interpret the rhythm

An 86-year-old patient with heart failure is receiving furosemide (lasix), 40mg I.V. the physician orders 40 mEq of potassium chloride in 100mml of dextrose 5% in water, to infuse over 4 hours. The patient's most recent serum potassium level is 3.0 MEq/L. At which infusion rate should the nurse set the I.V. pump? 25 ml/hour 10 ml/hour 100 ml/hour 50 ml/hour

25 mL / hour

The nurse coming on duty receives the report from the nurse going off duty. Which patient should the on-duty nurse assess first? A. The 58-year-old patient who was admitted 2 days ago with heart failure, blood pressure of 1126/76 mm Hg, and a respiratory rate of 22 breaths/minute The 89-year-old patient with end-stage right-sided heart failure, blood pressure of 78/50 mm Hg, and a "do not resuscitate" order. The 62-year-old patient who was admitted 1 day ago with thrombophlebitis and is receiving I.V. heparin. The 75-year old patient who was admitted 1 hour ago with new-onset atrial fibrillation and is receiving I.V. diltiazem (Cardizem).

A 75 year old admitted with new onset A FIB

Which rhythm pattern finding is indicative of PVC's? A QRS complex >0.12 second followed by a P wave Continuous wide QRS complexes with a ventricular rate of 160 bpm P waves hidden in QRS complexes with a regular rhythm of 120 bpm Saw-toothed P waves with no measurable PR interval and an irregular rhythm

A QRS > 0.12 followed by a P wave

During a visit to a 78-year-old with chronic heart failure, the home care nurse finds that the patient has ankle edema, a 2-kg weight gain over the past 2 days, and complains of "feeling too tired to get out of bed." Based on these data, the best nursing diagnosis for the patient is activity intolerance related to fatigue. disturbed body image related to weight gain. impaired skin integrity related to ankle edema. impaired gas exchange related to dyspnea on exertion.

Activity intolerance related to fatigue

A 34-year-old female with a history of asthma is admitted to the emergency department. The nurse notes that the patient is dyspneic, with a respiratory rate of 35 breaths/min, nasal flaring, and use of accessory muscles. Auscultation of the lung fields reveals greatly diminished breath sounds. Based on these findings, which action should the nurse take to initiate care of the client? Initiate oxygen therapy as prescribed and reassess the client in 10 minutes. Draw blood for an arterial blood gas. Encourage the patient to relax and breathe slowly through the mouth. Administer bronchodilators as prescribed.

Administer bronchodilators as prescribed

A 70-year-old patient is admitted to the hospital with a diagnosis of bacterial pneumonia. While obtaining the patient's health history, the nurse learns that the patient has osteoarthritis, follows a vegetarian diet, and is very concerned with cleanliness. Which of the following would most likely be a predisposing factor for the diagnosis of pneumonia? Age. Osteoarthritis Vegetarian diet. Daily bathing.

Age

A patient who has a history of chronic obstructive pulmonary disease (COPD) was hospitalized for increasing shortness of breath and chronic hypoxemia (SaO2 levels of 89% to 90%). In planning for discharge, which action by the nurse will be most effective in improving compliance with discharge teaching? Start giving the patient discharge teaching on the day of admission.. Have the patient repeat the instructions immediately after teaching. Accomplish the patient teaching just before the scheduled discharge. Arrange for the patient's caregiver to be present during the teaching.

Arrange for the patients caregiver to be present during the teaching

A patient's rhythm strip shows a regular rhythm with atrial and ventricular rates of 70 beats/minute, a PR interval of 0.24 seconds, and QRS duration of 0.08 seconds. The nurse interprets this rhythm as: Normal sinus rhythm (NSR) NSR with 1-degree atrioventricular (AV) block. Sinus arrhythmia. Accelerated junctional rhythm.

B NRS with 1 degree AV block

What is the earliest clinical manifestation in a patient with acute disseminated intravascular coagulation (DIC)? Severe shortness of breath. Bleeding without history or cause. Orthopnea. Hematuria.

Bleeding without history or cause

Good dental care is an important measure in reducing the risk of endocarditis. A teaching plan to promote good dental care in a patient with mitral stenosis should include instructing the patient to do which of the following? Select all that apply. Brush the teeth at least twice a day. Avoid use of an electric toothbrush. Take an antibiotic prior to oral surgery. Floss the teeth at least once a day. Have regular dental checkups.

Brush the teeth at least twice a day Floss the teeth at least once a day Have regular dental check ups

After a thoracotomy, the nurse instructs the patient to perform deep-breathing exercises. Which of the following is an expected outcome of these exercises? Deep breathing elevates the diaphragm, which enlarges the thorax and increases the lung surface available for gas exchange. Deep breathing increases blood flow to the lungs to allow them to recover from the trauma of surgery. Deep breathing controls the rate of air flow to the remaining lobe so that it will not become hyperinflated. Deep breathing expands the alveoli and increases the lung surface available for ventilation.

Deep breathing expands the alveoli and increases the lung surface bailable for ventilation

In the patient with a dysrhythmia, which assessment indicates decreased cardiac output (CO)? Hypertension and bradycardia Chest pain and decreased mentation Abdominal distention and hepatomegaly Bounding pulses and a ventricular heave

Chest pain and decrease mentation

A patient with severe chronic obstructive pulmonary disease (COPD) tells the nurse. "I wish I were dead! I'm just a burden on everybody." Based on this information, which nursing diagnosis is most appropriate? Complicated grieving related to expectation of death. Ineffective coping related to unknown outcome of illness Deficient knowledge related to lack of education about COPD Chronic low self-esteem related to increased physical dependence

Chronic low self esteem is related to increased physical dependence

The nurse assesses the respiratory status of a patient who is experiencing an exacerbation of chronic obstructive pulmonary disease (COPD) secondary to an upper respiratory tract infection. Which of the following findings would be expected? Normal breath sounds. Prolonged inspiration. Normal chest movement Coarse crackles and rhonchi

Coarse crackles and rhonchi

When assessing an individual with peripheral vascular disease, which clinical manifestation would indicate complete arterial obstruction in the lower left leg? Aching pain in the left calf. Burning pain in the left calf. Numbness and tingling in the left leg. Coldness of the left foot and ankle.

Coldness of the left food and ankle

Which of the following health promotion activities should the nurse include in the discharge teaching plan for a patient with asthma? Incorporate physical exercise as tolerated into the daily routine. Monitor peak flow numbers after meals and at bedtime. Eliminate stressors in the work and home environment. Use sedatives to ensure uninterrupted sleep at night.

Incorporate physical exercise as tolerated into the daily routine

While assessing a 68-year-old with ascites, the nurse also notes jugular venous distention (JVD) with the head of the patient's bed elevated 45 degrees. The nurse knows this finding indicates decreased fluid volume. jugular vein atherosclerosis. increased right atrial pressure. incompetent jugular vein valves.

Increase right arterial pressure

Which of the following physical assessment findings are normal for a patient with advanced chronic obstructive pulmonary disease (COPD)? Increased anteroposterior chest diameter. Underdeveloped neck muscles. Collapsed neck veins. Increased chest excursions with respiration.

Increased AP diameter

A hospitalized patient with heart failure suddenly develops dyspnea at rest, disorientation, confusion, and crackles in the lung bases on auscultation. The most important interventions by the nurse would be? Select all that apply. Insert a Foley catheter. Monitor urinary output Administer nasal oxygen. Administer a rapid-acting diuretic. Place the patient in a modified Trendelenburg position. Administer a 500ml intravenous normal saline bolus.

Inset foley Monitor urinary output Administer nasal O2 Administer rapid acting diuretic

The nurse is reviewing a patient's echocardiogram report, which states, "hypertrophy of the ventricular septum." The patient should be further evaluated for which type of cardiomyopathy? Congestive Dilated Hypertrophic Restrictive

Hypertrophic

One hour after I.V. furosemide (Lasix) is administered to a patient with heart failure, a short burst of ventricular tachycardia appears on the cardiac monitor. Which electrolyte imbalance should the nurse suspect? Hypocalcemia Hypermagnesemia Hypokalemia Hypernatremia

Hypokalemia

A patient is receiving Cilostazol (Pletal) for peripheral arterial disease causing intermittent claudication. The nurse determines this medication is effective when the patient reports which of the following? "I am having fewer aches and pains." "I do not have headaches anymore." "I am able to walk further without leg pain." "My toes are turning grayish black in color."

I am able to walk further without leg pain

The nurse is teaching a client who has been diagnosed with tuberculosis how to avoid spreading the disease to family members. Which statement(s) indicate(s) that the client has understood the nurse's instructions? Select all that apply. "I will need to dispose of my old clothing when i return home." "I should always cover my mouth and nose when sneezing." "It is important that I isolate myself from family when possible." "I should use paper tissues to cough in and dispose of them promptly."

I should always cover my mouth and nose when sneezing I should use paper tissues to cough in and dispose of them promptly

The nurse teaches a patient about pulmonary function testing (PFT). Which statement, if made by the patient, indicates teaching was effective? "I will use my inhaler right before the test." "I won't eat or drink anything 8 hours before the test." "I should inhale deeply and blow out as hard as I can during the test." "My blood pressure and pulse will be checked every 15 minutes after the test."

I should inhale deeply and blow out as hard as I can during the test

A lymph node biopsy is most often performed to diagnose Leukemia. Cause of lymphedema. Hemorrhagic tendencies Neoplastic cells in lymph nodes.

Neoplastic cells in the lymph nodes

The nurse is performing a cardiac assessment on her patient and auscultates a fourth heart sound (S4). The nurse interprets this as indicative of which of the following? Dilated aorta Normally functioning heart Decreased myocardial contractility Failure of the ventricle to eject all the blood during systole

Failure of the ventricle to eject all the blood during systole

A 55-year-old patient with increasing dyspnea is being evaluated for a possible diagnosis of chronic obstructive pulmonary disease (COPD). When teaching a patient about pulmonary function testing (PFT) for this condition, what is the most important question the nurse should ask? "Are you claustrophobic? "Are you allergic to shellfish?" "Do you have any metal implants or prostheses?" "Have you taken any bronchodilators in the past 6 hours?"

Have you taken any bronchodilators in the past 6 hours?

The nurse performs an assessment on a newly admitted client. The data include dyspnea, cough, weight gain, weakness, and edema. The nurse interprets these as signs and symptoms of: pericarditis hypertension. myocardial infarction (MI). heart failure.

Heart failure

Which of the following techniques for administering the Mantoux test is correct? Hold the needle and syringe almost parallel to the client's skin. Pinch the skin when inserting the needle. Aspirate before injecting the medication. Massage the site after injecting the medication.

Hold the needle and syringe almost parallel to the pt's skin

A patient who is experiencing an acute asthma attack is admitted to the emergency department. Which assessment should the nurse complete first? Listen to the patient's breath sounds. Ask about inhaled corticosteroid use. Determine when the dyspnea started. Obtain the forced expiratory volume (FEV) flow rate.

Listen to the patients breath sounds

A patient has mitral stenosis and will have a valve replacement. The nurse is instructing the patient about health maintenance prior to surgery. Inability to follow which of the following regimens would pose the greatest health hazard to this patient at this time? Medication therapy Diet modification Activity restrictions Dental care

Medication therapy

The nurse is assessing the lower extremities of the patient with peripheral vascular disease (PVD). During the assessment, the nurse should expect to find which of the following clinical manifestations of PVD? Select all that apply. Hairy legs. Mottled skin Pink skin Coolness Moist skin

Mottled skin Coolness

What is the rationale that supports multidrug treatment for patients with tuberculosis? Multiple drugs potentiate the drugs' actions. Multiple drugs reduce undesirable drug adverse effects. Multiple drugs allow reduced drug dosages to be given. Multiple drugs reduce development of resistant strains of the bacteria.

Multiple drugs reduce development of resistant strains of the bacteria

A patient hospitalized with chronic obstructive pulmonary disease (COPD) is being discharged home on oxygen therapy. Which instruction should the nurse include in the discharge teaching? Storage of oxygen tanks will require adequate space in the home. Travel opportunities will be limited because of the use of oxygen. Oxygen flow should be increased if the patient has more dyspnea. Oxygen use can improve the patient's prognosis and quality of life

O2 use can improve the patients prognosis and quality of life

The most effective measure the nurse can use to prevent wound infection when changing a patient's dressing after aortic valve surgery is to: Observe careful handwashing procedures. Clean the incisional area with an antiseptic. Use prepackaged sterile dressings to cover the incision. Place soiled dressings in a waterproof bag before disposing of them.

Observe careful hand washing procedures

Which nursing action for a patient with chronic obstructive pulmonary disease (COPD) could the nurse delegate to experienced unlicensed assistive personnel (UAP). Obtain oxygen saturation using pulse oximetry. Monitor for increased oxygen need with exercise. Teach the patient about safe use of oxygen at home. Adjust oxygen to keep saturation in prescribed parameters.

Obtain O2 sat using pulse ox

A patient with peripheral vascular disease has undergone a right femoral-popliteal bypass graft. The blood pressure has decreased from 124/80 to 94/62. What should the nurse assess first? IV fluid solution. Pedal pulses. Nasal cannula flow rate Capillary refill

Pedal pulses

The nurse teaches a client with chronic obstructive pulmonary disease (COPD) to assess for signs and symptoms of right-sided heart failure. Which of the following signs and symptoms should be included in the teaching plan? Clubbing of nail beds. Hypertension. Peripheral edema. Increased appetite.

Peripheral edema

A patient comes to the emergency department with acute shortness of breath and a cough that produces pink, frothy sputum. Admission assessment reveals crackles and wheezes, a blood pressure of 82/45 mm Hg, a heart rate of 120 beats/minute, and a respiratory rate of 38 breaths/minute. The patient's medical history includes diabetes mellitus, hypertension, and heart failure. Which disorder should the nurse suspect? Pulmonary edema Pneumothorax Cardiac tamponade Pulmonary embolus

Pulmonary edema

Which statement by a nurse to the health care aide best explains the need to promptly report changes in respiratory rate for a patient diagnosed with heart failure? "Pulmonary edema, a life-threatening condition, can develop in minutes." "Severe acute respiratory syndrome (SARS) is a complication of heart failure." "Pneumonia is a consequence of inadequate ventilation with heart failure." "Pneumothorax, a life-threatening condition, can develop in minutes."

Pulmonary edema, a life threatening condition, can develop in minutes

In which of the following positions should the nurse place a patient with heart failure who has orthopnea? Semisitting (low Fowler's position) with legs elevated on pillows. Lying on the right side (Sims' position) with a pillow between the legs. Sitting upright (high Fowler's position) with legs resting on the mattress. Lying on the back with the head lowered (Trendelenburg's position) and legs elevated.

Sitting upright (high fowlers) with legs resting on the mattress

You are evaluating the 12-lead ECG of a patient experiencing an inferior wall myocardial infarction (MI). While conferring with the health care team, which ECG changes associated with an evolving MI do you correctly identify? Select all that apply. Notched T wave Presence of a U wave T-wave inversion D. Prolonged PT interval ST-segment elevation Pathologic Q wave

T wave inversion ST Segment election Pathologic Q Wave

The nurse analyzes the results of a patient's arterial blood gases (ABGs). Which finding would require immediate action? The bicarbonate level (HCO3) is 31 mEq/L. The arterial oxygen saturation (SaO2) is 92%. The partial pressure of CO2 in arterial blood (PaCO2) is 31 mm Hg. The partial pressure of oxygen in arterial blood (Pa02) is 59 mm Hg.

The partial pressure of O2 in arterial blood is 59

Which of the following indicates that the patient with chronic obstructive pulmonary disease (COPD) who has been discharged to home understands the care plan? The patient promises to do pursed-lip breathing at home. The patient states actions to reduce pain. The patient will use oxygen via a nasal cannula at 5 L/min The patient agrees to call the physician if dyspnea on exertion increases.

The patient agrees to call the physician if dyspnea on exertion increases

The nurse takes an admission history on a patient with possible asthma who has new-onset wheezing and shortness of breath. Which information may indicate a need for a change in therapy? The patient has chronic inflammatory bowel disease. The patient has a history of pneumonia 6 months ago. The patient takes propranolol (Inderal) for hypertension. The patient uses acetaminophen (Tylenol) for headaches.

The patient takes propranolol for HTN

Which information will the nurse include in the asthma teaching plan for a patient being discharged? Use the inhaled corticosteroid when shortness of breath occurs. Inhale slowly and deeply when using the dry powder inhaler (DPI). Hold your breath for 5 seconds after using the bronchodilator inhaler. Tremors are an expected side effect of rapidly acting bronchodilators

Tremors are an expected side effect of rapid acting bronchodilators

The patient with peripheral vascular disease has been prescribed diltiazem (Cardizem). The nurse should determine the effectiveness of this medication by assessing the patient for: Relief of anxiety Sedation Vasoconstriction Vasodilation

Vasodilation

The nurse should teach the patient with asthma that which of the following is one of the most common precipitating factors of an acute asthma attack? Occupational exposure to toxins. Viral respiratory infections. Exposure to cigarette smoke. Exercising in cold temperatures.

Viral respiratory infections

A patient was admitted to the hospital with iron deficiency anemia and blood-streaked emesis. Which question is most appropriate for the nurse to ask in determining the extent of the patient's activity intolerance? "What daily activities were you able to do 6 months ago compared with the present?" "How long have you had this problem?" "Have you been able to keep up with all your usual activities?" "Are you more tired now than you used to be?"

What daily activities were you able to do 6 months ago compared with the present?

When a patient with thrombocytopenia has a severe headache, the nurse interprets that this may indicate which of the following? Stress of the disease. Cerebral bleeding. Migraine headache. Sinus congestion.

cerebral bleeding

The nurse should instruct a young female adult with sickle cell anemia to do which of the following? Select all that apply. Drink plenty of fluids when outside in hot weather. Avoid travel to cities where the oxygen level is lower. Be aware that since she is homozygous for HbS, she carries the sickle cell trait. Know that pregnancy with sickle cell disease increases the risk of a crisis. Avoid flying on commercial airlines.

drink plenty of fluids avoid travel to cities with O2 level is lower Know that pregnancy with sickle cell disease increases the risk of crisis

When caring for a patient with a chest tube and water-seal drainage system, the nurse should: Verify that the air vent on the water-seal drainage system is capped when the suction is off. Strip the chest drainage tubes at least every 4 hours if excessive bleeding occurs. Ensure that the chest tube is clamped when moving the patient out of the bed. Make sure that the drainage apparatus is always below the patient's chest level.

make sure that the drainage apparatus is always below the patients chest level

A patient with pneumonia is experiencing pleuritic chest pain. The nurse should assess the patient for: A mild but constant aching in the chest. Severe midsternal pain. Moderate pain that worsens on inspiration. Muscle spasm pain that accompanies coughing.

moderate pain that worsens on inspiration


Related study sets

Chapter 19 Program Design and Technique for Speed and Agility Training

View Set

TOGAF 9.2 Questions for Review - v1

View Set

Chapter 16: Workers' Compensation Insurance

View Set

Chapter 5: Mental Status Assessment

View Set